You are on page 1of 62

MISCELLANEOUS

1. To determine the greatest possible number of contributors we must assume that each
of these individuals contributed the minimum amount, or $50. We can then set up an
inequality in which n equals the number of contributors
50n is less than or equal to $1,!"#
$ivide both sides of the equation by 50 to isolate n, and get
n is less than or equal to %".#&
'ince n represents individual people, it must be the greatest whole number less than
%".#&. Thus, the greatest possible value of n is %".
(lternately, we could have assumed that the fundraiser collected $1,!50 rather than
$1,!"#. )f it had, and we assumed each individual contributed the minimum amount,
there would have been e*actly %5 contributors +$50 * %5 , $1,!50-. 'ince the fundraiser
actually raised one dollar less than $1,!50, there must have been one fewer contributor, or
%".
The correct answer is ..
/.
)t may be easiest to represent the ages of 0oan, 1ylie, 2illian and 3iriam +J, K, L and M-
on a number line. )f we do so, we will see that the ages represent consecutive integers as
shown in the diagram.
'ince the ages are consecutive integers, they can all be e*pressed in terms of L L, L 4 1,
L 4 /, L 4 %. The sum of the four ages then would be "L 4 5. 'ince L must be an integer
+it6s 2illian6s age-, the e*pression "L 4 5 describes a number that is two more than a
multiple of "
"L 4 5 , +"L 4 "- 4 /
7"L 4 " describes a multiple of ", since it can be factored into "+L 4 1- or " 8 an integer.9
5" is the only number in the answer choices that is two more than a multiple of " +namely,
5/-.
The correct answer is $.
%. This is an algebraic translation problem dealing with ages. :or this type of problem,
an age chart can help us ;eep trac; of the variables
<=W )< 5 >?(@'
0(<?T J J 4 5
A(@=2 C C 4 5
Bsing the chart in combination with the statements given in the question, we can derive
equations to relate the variables. The first statement tells us that 0anet is now /5 years
younger than her mother Aarol. 'ince we have used J to represent 0anet6s current age, and
C to represent Aarol6s current age, we can translate the statement as follows J , C C /5.
The second statement tells us that 0anet will be half Aarol6s age in 5 years. 'ince we have
used (J 4 5- to represent 0anet6s age in 5 years, and +C 4 5- to represent Aarol6s age in 5
years, we can translate the statement as follows J 4 5 , +1D/-+C 4 5-.
<ow, we can substitute the e*pression for C +C , J 4 /5- derived from the first equation
into the second equation +note we choose to substitute for C and solve for J because the
question as;s us for 0anetEs age 5 years ago-
J 4 5 , +1D/-+J 4 /5 4 5-
J + 6 , +1D/-+J 4 %1-
/J 4 1/ , J 4 %1
J , 1#
)f 0anet is now 1# years old, she was 1" years old 5 years ago.
The correct answer is ..
".
The $1,""0 is divided into 1/ equal monthly allocations.
1""0D1/ , $1/0
The company has $1/0 allocated per month for entertainment, so the allocation for three
months is 1/0 F % , %50
'ince the company has spend a total of $%00 thus far, it is $%50 G $%00 , $50 under
budget.
The correct answer is (.
5.
'ince this problem includes variables in both the question and the answer choices, we can
try solving by plugging in smart numbers. :or *, we want to choose a multiple of /
because we will have to ta;e *D/ later. 2etEs say that (A3? produces " brooms per month
from 0anuary to (pril, so * , ". The total number of brooms produced was +" brooms * "
months-, or 15 brooms.
(A3? sold *D/ brooms per month, or / brooms per month +because we chose * , "-.
<ow we need to start figuring out the storage costs from 3ay /
nd
to $ecember %1
st
. 'ince
(A3? sold / brooms on 3ay 1
st
, it needed to store 1" brooms that month, at a cost of
$1". :ollowing the same logic, we see that (A3? sold another two brooms 0une 1
st
and
stored 1/ brooms, which cost the company $1/. We now see that the 0uly storage costs
were $10, (ugust were $&, 'eptember $5, =ctober $", <ovember $/, and for $ecember
there were no storage costs since the last / brooms were sold on $ecember 1
st
.
'o (A3?Es total storage costs were 1" 4 1/ 4 10 4 & 4 5 4 " 4 / , $55. <ow we Hust
need to find the answer choice that gives us $55 when we plug in the same value, * , ",
that we used in the question. 'ince 1" * " , 55, $1"* must be the correct value.
The correct answer is ?.
While plugging in smart numbers is the preferred method for I)A problems such as this
one, it is not the only method. .elow is an alternative, algebraic method for solving this
problem
(A3? accumulated an inventory of "x brooms during its fourGmonth production period.
)f it sold 0.5x brooms on 3ay 1
st
, then it paid storage for %.5x brooms in 3ay, or $%.5x.
(gain, if (A3? sold 0.5x brooms on 0une 1
st
, it paid storage for %x brooms in 0une, or
$%x. The first row of the table below shows the amount of money spent per month on
storage. <otice that since (A3? liquidated its stoc; on $ecember 1
st
, it paid Jero dollars
for storage in $ecember.
3(> 0B< 0B2 (BK '?L =AT <=I
$%.5x $%x $/.5x $/x $1.5x $1x $0.5x
)f we add up these costs, we see that (A3? paid $1"x for storage.
5.
The bus will carry its greatest passenger load when P is at its ma*imum value. )f P , G/+S
C "-
/
4 %/, the ma*imum value of P is %/ because +S C "-
/
will never be negative, so the
e*pression G/+S C "-
/
will never be positive. The ma*imum value for P will occur when
G/+S C "-
/
, 0, i.e. when S , ".
The question as;s for the number of passengers two stops after the bus reaches its greatest
passenger load, i.e. after 5 stops +S , 5-.
P , G/+5 C "-
/
4 %/
P , G/+/-
/
4 %/
P , G& 4 %/
P , /"
The correct answer is A.
(lternatively, the ma*imum value for P can be found by building a table, as follows
S P
0 0
1 1"
/ /"
% %0
" %/
5 %0
5 /"
The ma*imum value for P occurs when S , ". Thus, two stops later at S , 5, P , /".
(nswer choice A is correct.
!. 0ohn was /! when he married .etty, and since they Hust celebrated their fifth wedding
anniversary, he is now %/.
'ince .ettyEs age now is !D& of 0ohnEs, her current age is +!D&- F %/, which equals /&.
The correct answer is A.
&. 0oe uses 1D" of %50, or #0 gallons, during the first wee;. Me has /!0 gallons
remaining +%50 C#0 , /!0-.
$uring the second wee;, 0oe uses 1D5 of the remaining /!0 gallons, which is 5" gallons.
Therefore, 0oe has used 1"" gallons of paint by the end of the second wee; +#0 4 5" ,
1""-.
The correct answer is ..
#. =ne way to do this problem is to recogniJe that the star earned $&3 more +$%/3 G
$/"3 , $&3- when her film grossed $"03 more +$1003 G $503 , $"03-. 'he
wants to earn $"03 on her ne*t film, or $&3 more than she earned on the more
lucrative of her other two films. Thus, her ne*t film would need to gross $"03 more
than $1003, or $1"03.
(lternatively, we can solve this problem using algebra. The starEs salary consists of a
fi*ed amount and a variable amount, which is dependent on the gross revenue of the
film. We ;now what she earned for two films, so we can set up two equations, where f
is her fi*ed salary and p is her portion of the gross, e*pressed as a decimal
'he earned $%/ million on a film that grossed $100 million $%/3 , f 4 p+$1003-
'he earned $/" million on a film that grossed $50 million $/"3 , f 4 p+$503-
We can solve for p by subtracting the second equation from the first
$%/3 , f 4 p+$1003-
C 7$/"3 , f 4 p +$503-9
$&3 , p+$"03-
0./ , p
We can now plug in 0./ for p in either of the original equations to solve for f
$%/3 , f 4 0./+$1003-
$%/3 , f 4 $/03
$1/3 , f
<ow that we ;now her fi*ed salary and the percentage of the gross earnings she
receives, we can rewrite the formula for her total earnings as
Total earnings , $1/3 4 0./+gross-
:inally, we Hust need to figure out how much gross revenue her ne*t film needs to
generate in order for her earnings to be $"0 million
$"03 , $1/3 4 0./+gross-
$/&3 , 0./+gross-
$/&3D0./ , $1"03 , gross
The correct answer is $.
10. ). B<A?@T()< )t depends on how many bicycles <orman sold.
:or e*ample, if x , ", then <orman earned $"" 7, $/0 4 +" F $5-9 last wee;. )n order to
double his earnings, he would have to sell a minimum of # bicycles this wee; +y ,
#-, ma;ing $#/ 7, $/0 4 +5 F $5- 4 +% F $1/-9. )n that case, y N /x.
Mowever, if x , 5 and y , 11, then <orman would have earned $55 7, $/0 4 +5 F $5-9 last
wee; and $115 7, $/0 4 +5 F $5- 4 +5 F $1/-9 this wee;. )n that case, $115 N / F $55, yet
y O /x.
'o, it is possible for <orman to more than double his earnings without selling twice as
many bicycles.
)). T@B? )n order to earn more money this wee;, <orman must sell more bicycles.
))). T@B? )f <orman did not sell any bicycles at all last wee; +x , 0-, then he would
have earned the minimum fi*ed salary of $/0. 'o he must have earned at least $"0 this
wee;. )f y , %, then <orman earned $%& 7, $/0 4 +% F $5-9 this wee;. )f y , ", then
<orman earned $"" 7, $/0 4 +" F $5-9 this wee;. Therefore, <orman must have sold at
least " bicycles this wee;, which can be e*pressed y N %.
The correct answer is $.
11. )n order to determine the greatest number of points that an individual player might
have scored, assume that 11 of the 1/ players scored ! points, the minimum possible.
The 11 low scorers would therefore account for !+11- , !! points out of 100. The
number of points scored by the twelfth player in this scenario would be 100 C !! , /%.
The correct answer is ?.
1/. 'ince we are not given any actual spending limits, we can pic; numbers. )n problems
involving fractions, it is best to pic; numbers that are multiples of the denominators.
We can set the spending limit for the gold account at $15, and for the platinum card at
$%0. )n this case, 'ally is carrying a balance of $5 +which is 1D% of $15- on her gold
card, and a balance of $5 +1D5 of $%0- on her platinum card. )f she transfers the
balance from her gold card to her platinum card, the platinum card will have a balance
of $11. That means that $1# out of her $%0 spending limit would remain unspent.
(lternatively, we can solve this algebraically by using the variable x to represent the
spending limit on her platinum card
+1D5-x 4 +1D%-+1D/-x ,
+1D5-x 4 +1D5-x ,
+5D%0-x 4 +5D%0-x ,
+11D%0-x
This leaves 1#D%0 of her spending limit untouched.
The correct answer is $.
13.
The problem tal;s about 3artina and LamEs incomes but never provides an actual dollar
value, either in the question or in the answer choices. We can, therefore, use smart
numbers to solve the problem. .ecause the dollar value is unspecified, we pic; a dollar
value with which to solve the problem. To answer the question, we need to calculate
dollar values for the portion of income each earns during the ten months not including
0une and (ugust, and we also need to calculate dollar values for each playerEs annual
income.
2etEs start with 3artina, who earns 1D5 of her income in 0une and 1D& in (ugust. The
common denominator of the two fractions is /", so we set 3artinaEs annual income at
$/". This means that she earns $" +1D5 F /"- in 0une and $% +1D& F /"- in (ugust, for a
total of $! for the two months. )f 3artina earns $! of $/" in 0une and (ugust, then she
earns $1! during the other ten months of the year.
The problem tells us that Lam earns the same dollar amount during the two months as
3artina does, so Lam also earns $! for 0une and (ugust. The $! Lam earns in 0une and
(ugust represents 1D% 4 1D" of her annual income. To calculate her annual income, we
solve the equation ! , +1D% 4 1D"-x, with x representing LamEs annual income. This
simplifies to ! , +!D1/-x or 1/ , x. )f Lam earns $! of $1/ in 0une and (ugust, then she
earns $5 during the other ten months of the year. 7<=T? we cannot simply pic; a
number for Lam in the same way we did for 3artina because we are given a relationship
between 3artinaEs income and LamEs income. )t is a coincidence that LamEs income of
$1/ matches the common denominator of the two fractions assigned to Lam, 1D% and 1D" G
if we had pic;ed $"& for 3artinaEs income, LamEs income would then have to be $/", not
$1/.9
Aombined, the two players earn $1! 4 $5 , $// during the other ten months, out of a
combined annual income of $/" 4 $1/ , $%5. The portion of the combined income earned
during the other ten months, therefore, is //D%5 which simplifies to 11D1&.
<ote first that you can also calculate the portion of income earned during 0une and
(ugust and then subtract this fraction from 1. The portion of income earned during 0une
and (ugust, !D1&, appears as an answer choice, so be careful if you decide to solve it this
way.
<ote also that simply adding the four fractions given in the problem produces the number
!D&, an answer choice. 1D& +or 1 C !D&- is also an answer choice. These two answers are
Ptoo good to be trueP G that is, it is too easy to arrive at these numbers.
The correct answer is $.
1". This fraction problem contains an PunspecifiedP total +the x liters of water in the la;e-.
Lic; an easy PsmartP number to ma;e this problem easier. Bsually, the smart number
is the lowest common denominator of all the fractions in the problem. Mowever, if
you pic; /&, you will quic;ly see that this yields some unwieldy computation.
The easiest number to wor; with in this problem is the number ". 2etEs say there are "
liters of water originally in the la;e. The question then becomes $uring which year is the
la;e reduced to less than 1 liter of waterQ
(t the end of /0!5, there are " F +5D!- or /0D! liters of water in the la;e. This is not less
than 1.
(t the end of /0!!, there are +/0D!- F +5D!- or 100D"# liters of water in the la;e. This is
not less than 1.
(t the end of /0!&, there are +100D"#- F +5D!- or 500D%"% liters of water in the la;e. This
is not less than 1.
(t the end of /0!#, there are +500D%"%- F +5D!- or /500D/"01 liters of water in the la;e.
This is not less than 1.
(t the end of /0&0, there are +/500D/"01- F +5D!- or 1/500D15&0! liters of water in the
la;e. This is less than 1.
<otice that pic;ing the number " is essential to minimiJing the computation
involved, since it is very easy to see when a fraction falls below 1 +when the numerator
becomes less than the denominator.- The only moderately difficult computation involved
is multiplying the denominator by ! for each new year.
The correct answer is $.
15. This fraction problem contains an unspecified total +the number of married couples-
and is most easily solved by a pic;ing a PsmartP number for that total. The smart number
is the least common denominator of all the fractions in the problem. )n this case, the
smart number is /0.
2etEs say there are /0 married couples.
15 couples +%D" of the total- have more than one child.
& couples +/D5 of the total- have more than three children.
This means that 15 C & , ! couples have either / or % children. Thus !D/0 of the married
couples have either / or % children.
The correct answer is A.
15. We can bac; solve this question by using the answer choices. 2et6s first chec; to
ma;e sure that each of the 5 possible prices for one candy can be paid using e*actly "
coins
& , 5414141
1% , 10414141
"0 , 10410410410
5% , 50414141
55 , 504104541
'o far we can6t ma;e any eliminations. <ow let6s chec; two pieces of candy
15 , 5 4 5 4 5 4 1
/5 , 10 4 10 4 5 4 1
&0 , /5 4 /5 4 /5 4 5
105 , 50 4 50 4 5 4 1
1%/ , 50 4 50 4 /5 4 5 4 1 4 1
We can eliminate answer choice ? here. <ow three pieces of candy
/" , 10 4 10 4 1 4 1 4 1 4 1
%# , /5 4 10 4 1 4 1 4 1 4 1
1/0 , 50 4 50 4 10 4 10
15# , 50 4 50 4 50 4 5 4 1 4 1 4 1 4 1.
We can eliminate answer choices (, . and $.
<otice that at a price of "0R, .illy can buy four and five candies with e*actly " coins as
well
150 , 50 4 50 4 50 4 10
/00 , 50 4 50 4 50 4 50
This problem could also have been solved using divisibility and remainders. <otice that
all of the coins are multiples of 5 e*cept pennies. )n order to be able to pay for a certain
number of candies with e*actly four coins, the total price of the candies cannot be a value
that can be e*pressed as 5x 4 ", where x is a positive integer. )n other words, the total
price cannot be a number that has a remainder of " when divided by 5. WhyQ The
remainder of " would alone require " pennies.
We can loo; at the answer choices now Hust focusing on the remainder when each price
and its multiples are divided by 5
Lrice per
candy
@emainder
when price
for
1 candy is
divided by 5
@emainder
when price
for
/ candies is
divided by 5
@emainder
when price
for
% candies is
divided by 5
@emainder
when price
for
" candies is
divided by 5
& % 1 " 1
1% % 1 " /
"0 0 0 0 0
5% % 1 " /
55 1 / % "
The only price for which none of its multiples have a remainder of " when divided by 5 is
"0R.
<otice that not having a remainder of " does not guarantee that e*actly four coins can be
usedS however, having a remainder of " does guarantee that e*actly for coins cannot be
usedT
The correct answer is A.
17.
:rom the question we ;now that "0 percent of the violetDgreen mi* is blue pigment. We
also ;now that %0 percent of the violet paint and 50 percent of the green paint is blue
pigment. 'ince the blue pigment in the violetDgreen mi* is the same blue pigment in the
original violet and green paints, we can construct the following equation
.%v 4 .5g , ."+v + g-
.%v 4 .5g , ."v 4 ."g
.1g , .1v
g v
Therefore, the amount of violet paint is equal to the amount of green paint in the brown
mi*ture, each contributing 50 percent of the total. 'ince the red pigment represents !0
percent of the weight of the violet paint, it must account for !0 percent of 50 percent of
the weight of the brown mi*. This represents +.!-+.5- , .%5, or %5U of the total weight of
the brown mi*. 'ince we have 10 grams of the brown paint, the red pigment must account
for +.%5-+10- , %.5 grams of the brown paint.
There is an alternative way to come up with the conclusion that there must be equal
amounts of green and violet paints in the mi*. 'ince there is blue paint in both the violet
and green paints, when we combine the two paints, the percentage of blue paint in the mi*
will be a !eighted average of the percentages of blue in the violet paint and the
percentage of blue in the green paint. :or e*ample, if there is twice as much violet as
green in the brown mi*, the percentage of blue in the violet will get double weighted.
:rom loo;ing at the numbers, however, "0U is e*actly the simple average of the %0U
blue in violet and the 50U blue in green. This means that there must be an equal amount
of both paints in the mi*.
'ince there are equal amounts of violet and green paint in the 10 grams of brown mi*ture,
there must be 5 grams of each. The violet paint is !0U red, so there must be +.!-+5- , %.5
grams of red paint in the mi*.
The correct answer is ..
18.
This question requires us to untangle a series of ratios among the numbers of wor;ers in
the various years in order to find the number of wor;ers after the first year. We can solve
this problem by setting up a grid to ;eep trac; of the information
.efore (fter >ear 1 (fter >ear / (fter >ear % (fter >ear "
We are told initially that after the fourGyear period, the company has 10,500 employees
.efore (fter >ear 1 (fter >ear / (fter >ear % (fter >ear "
10,500
We are then told that the ratio of the number of wor;ers after the fourth year to the
number of wor;ers after the second year is 5 to 1. This implies that the number of
wor;ers after the fourth year is si* times greater than that after the second year. Thus the
number of wor;ers after the second year must be 10,500D5 , 1,!50
.efore (fter >ear 1 (fter >ear / (fter >ear % (fter >ear "
1,!50 10,500
We are then told that the ratio of the number of wor;ers after the third year to the number
after the first year is 1" to 1. We can incorporate this into the chart
.efore (fter >ear 1 (fter >ear / (fter >ear % (fter >ear "
x 1,!50 1"x 10,500
<ow we are told that the ratio of the number of wor;ers after the third year to that before
the period began is !0 to 1. We can incorporate this into the chart as well
.efore (fter >ear 1 (fter >ear / (fter >ear % (fter >ear "
" x 1,!50
1"x
!0y
10,500
:rom the chart we can see that 1"x , !0y# Thus x , 5y
.efore (fter >ear 1 (fter >ear / (fter >ear % (fter >ear "
" $y 1,!50 !0y 10,500
'ince the ratio between consecutive years is always an integer and since after three years
the number of wor;ers is !0 times greater, we ;now that the series of ratios for the first
three years must include a /, a 5, and a ! +because / * 5 * ! , !0-. .ut this fact by itself
does not tell us the order of the ratios. )n other words, is it / G 5 G ! or ! G / G 5 or 5 G / G !,
etcQ We do ;now, however, that the factor of 5 is accounted for in the first year. 'o we
need to ;now whether the number of wor;ers in the second year is twice as many or
seven times as many as in the first year.
@ecall that the number of wor;ers after the fourth year is si* times greater than that after
the second year. This implies that the ratios for the third and fourth years must be / and %
or % and /. This in turn implies that the ratio of ! to 1 must be between the first and
second years. 'o 1,!50 is ! times greater than the number of wor;ers after the first year.
Thus, 1,!50D! , /50.
(lternatively, since the question states that the ratio between any two years is always an
integer, we ;now that 1,!50 must be a multiple of the number of wor;ers after the first
year. 'ince only !0 and /50 are factors of 1!50, we ;now the answer must be either
choice . or choice A. )f we assume that the number of wor;ers after the first year is !0,
however, we can see that this must cannot wor;. The number of wor;ers always increases
from year to year, but if !0 is the number of wor;ers after the first year and if the number
of wor;ers after the third year is 1" times greater than that, the number of wor;ers after
the third year would be 1" * !0 , #&0, which is less than the number of wor;ers after the
second year. 'o choice . is eliminated and the answer must be choice A.
The correct answer is choice A /50.
19.
)t is important to remember that if the ratio of one group to another is xy, the total
number of obHects in the two groups together must be a multiple of x 4 y. 'o since the
ratio of rams to ewes on the farm is " to 5, the total number of sheep must be a multiple of
# +" parts plus 5 parts-. (nd since the ratio of rams to ewes in the first pen is " to 11, the
total number of sheep in the first pen must be a multiple of 15 +" parts plus 11
parts-. 'ince the number of sheep in each pen is the same, the total number of sheep must
be a multiple of both # and 15.
)f we assume that the total number of sheep is "5 +the lowest common multiple of # and
15-, the number of rams is /0 and the number of ewes is /5 +ratio "5-.
"5D% , 15, so there are 15 sheep in each pen. Therefore, there are " rams and 11 ewes in
the first pen +ratio "11-. This leaves /0 G " , 15 rams and /5 G 11 , 1" ewes in the other
two pens. 'ince the second and third pens have the same ratio of rams to ewes, they must
have 15D/ , & rams and 1"D/ , ! ewes each, for a ratio of &! or &D!.
(lternatively, we can answer the question algebraically.
'ince the ratio of rams to ewes in the first pen is "11, let the number of rams in the
first pen be "x and the number of ewes be 11x. 2et r be the number of rams in the
second pen and let e be the number of ewes in the second pen. 'ince the number of sheep
in each pen is the same, we can construct the following equation "x 4 11x , r 4 e, or 15x
, r 4 e.
'ince the number of sheep in each pen is the same, we ;now that the number of rams in
the second and third pens together is /r and the number of ewes in the second and third
pens together is /e. Therefore, the total number of rams is "x 4 /r. The total number of
ewes is 11x 4 /e. 'ince the overall ratio of rams to ewes on the farm is "5, we can
construct and simplify the following equation
We can find the ratio of r to e by setting the equations we have equal to each other. :irst,
though, we must multiply each one by coefficients to ma;e them equal the same value
'ince both equations now equal 1/0x, we can set them equal to each other and simplify
The correct answer is (.
20.
We can find a ratio between the rates of increase and decrease for the corn and wheat
To get rid of the radical sign in the denominator, we can multiply top and bottom by
and simplify
This ratio indicates that for every cent that the price of wheat decreases, the price of corn
increases by cents. 'o if the price of wheat decreases by x cents, the price of
corn will increase by cents.
'ince the difference in price between a pec; of wheat and a bushel of corn is currently
$/.50 or /50 cents, the amount by which the price of corn increases plus the amount by
which the price of wheat decreases must equal /50 cents. We can e*press this as an
equation
%&ount Corn 'n(reases 4 %&ount )heat *e(reases , /50
We can then rewrite this word equation using variables. 2et ( be the decrease in the price
of wheat in cents
<otice that the radical / was replaced with its appro*imate numerical value of 1."
because the question as;s for the appro*imate price. We need not be e*act in this
particular instance.
)f ( , /0, we ;now that the price of a pec; of wheat had decreased by /0 cents when it
reached the same level as the increased price of a bushel of corn. 'ince the original price
of a pec; of wheat was $5.&0, its decreased price is $5.&0 G $0./0 , $5.50.
+.y the same to;en, since ( , /0, the price of a bushel of corn had increased by /0+
- cents when it reached the same level as the decreased price of a pec; of wheat.
This is equivalent to an increase of appro*imately /"0 cents. Thus the increased price of a
bushel of corn , $%./0 4 $/."0 , $5.50.-
The correct answer is ?.
/1.
2et s represent the number of science maHors, & represent the number of math maHors, h
represent the number of history maHors, and l represent the number of linguistics maHors.
We can set up the following equations
s , +1D%+h
& , +/D%-h
s 4 & 4 h 4 l , /000
We can substitute and isolate the number of linguistics maHors.
+1D%+h 4 +/D%+h 4 h 4 l , /000
/h 4 l , /000
l, /000 C /h
We can rephrase the question PMow many students maHor in historyQP
+1- 'B::)A)?<T )f l , &, and & , +/D%-h, we can solve for h
+1D%-h 4 +/D%-h 4 h 4 l , /000
+1D%+h 4 +/D%+h 4 h 4+/D%-h , /000
+&D%-h , /000
h , /000+%D&-
h , !50
)f h , !50, l , +/D%+h , 500.
+/- 'B::)A)?<T )f & , s 4 /50, and & , +/D%-h and s , +1D%-h, we can substitute and
solve for h
+/D%-h , +1D%-h 4 /50
+1D%+h , /50
h , !50
)f h , !50, l , /000 C /+!50- , 500.
The correct answer is $.
22.
We can thin; of the liquids in the red buc;et as liquids (, ., A and ?, where ? represents
the totality of every other ,ind of li-uid that is not %. /. or C# )n order to determine the
percentage of ? contained in the red buc;et, we will need to determine the total amount of
( 4 . 4 A and the total amount of ?.
't is T0MPT'12 (3ut in(orre(t+ to use the follo!ing logi( !ith the infor&ation given in
State&ent (1+#
'tatement +1- tells us that the total amount of liquids (, ., and A now in the red buc;et is
1./5 times the total amount of liquids ( and . initially contained in the green buc;et.
2etEs begin by assuming that, initially, there are 10 ml of liquid ( in the green buc;et.
Bsing the percentages given in the problem we can now determine that the composition
of the green buc;et was as follows
10U ( , 10 ml
10U . , 10 ml
&0U ? , &0 ml
'ince there were /0 total ml of ( and . in the green buc;et, we ;now from statement +1-
that there must be /5 ml of ( 4 . 4 A now in the red buc;et +since /5 is 1./5 times /0-.
:rom this we can deduce that, there must have been 5 ml of A in the blue buc;et. We can
use the percentages given in the problem to determine the e*act initial composition of the
blue buc;et
10U A , 5 ml
#0U ? , "5 ml
'ince the liquid in the red buc;et is simply the totality of all the liquids in the green
buc;et plus all the liquids in the blue buc;et, we can use this information to determine the
total amount of ( 4 . 4 A +/5 ml- and the total amount of ? +&0 4 "5 , 1/5 ml- in the red
buc;et. Thus, the percentage of liquid now in the red buc;et that is <=T (, ., or A is
equal to 1/5D150 , &% 1D% percent.
This ratio +or percentage- will always remain the same no matter what initial amount we
choose for liquid ( in the green buc;et. This is because the relative percentages are fi*ed.
We can generaliJe that given an initial amount * for liquid ( in the green buc;et, we
;now that the amount of liquid . in the green buc;et must also be * and that the amount
of ? in the green buc;et must be &*. We also ;now that the amount of liquid A in the blue
buc;et must be .5*, which means that the amount of ? in the blue buc;et must be ".5*.
Thus the total amount of ( 4 . 4 A in the red buc;et is * 4 * 4 .5* , /.5* and the total
amount of liquid ? in the red buc;et is &* 4 ".5* , 1/.5*. Thus the percentage of liquid
now in the red buc;et that is <=T (, ., or A is equal to 1/.5*D15* or &% 1D% percent.
4o!ever. the a3ove logi( is 5L%)0* 3e(ause it assu&es that the green 3u(,et does not
(ontain li-uid C and that the 3lue 3u(,et does not (ontain li-uids % or /#
)n other words, the above logic assumes that ;nowing that there are * ml of ( in the green
buc;et implies that there are &* ml of ? in the green buc;et. @emember, however, that ?
is defined as the totality of every li-uid that is 16T %. /. or C7 While the problem gives
us information about the percentages of ( and . contained in the green buc;et, it does not
tell us anything about the percentage of A contained in the green buc;et and we cannot
Hust assume that this is 0. )f the percentage of A in the green buc;et is not 0, then this will
change the percentage of ? in the green buc;et as well as changing the relative amount of
liquid A in the blue buc;et.
:or e*ample, letEs say that the green buc;et contains 10 ml of liquids ( and . but also
contains % ml of liquid A. Ta;e a loo; at how this changes the logic
Kreen buc;et
10U ( , 10 ml
10U . , 10 ml
%U A , % ml
!!U ? , !! ml
'ince there are /0 total ml of ( and . in the green buc;et, we ;now from statement +1-
that there must be /5 ml of ( 4 . 4 A in the red buc;et +since /5 is 1./5 times /0-.
'ince the green buc;et already contributes /% ml of this total, we ;now that there must be
/ total ml of liquids (, . and A in the blue buc;et. )f the blue buc;et does not contain
liquids ( or . +which we cannot necessarily assume-, then the composition of the blue
buc;et would be the following
10U A , / ml
#0U ? , 1& ml
<ote, however, that if the blue buc;et does contain some of liquids ( or ., then the
composition of the blue buc;et might also be the following
10U A , 1 ml
10U ( , 1 ml
&0U ? , & ml
<otice that it is impossible to ascertain the e*act amount of ? in the red buc;et G since this
amount will change depending on whether the green buc;et contains liquid A andDor the
blue buc;et contains liquids ( or ..
Thus statement +1- by itself is <=T sufficient to answer this question.
'tatement +/- tells us that the green and blue buc;ets did not contain any of the same
liquids. (s such, we ;now that the green buc;et did not contain liquid A and that the blue
buc;et did not contain liquids ( or .. =n its own, this does not help us to answer the
question. Mowever, ta;ing 'tatement +/- together with 'tatement +1-, we can definitively
answer the question.
The correct answer is A .=TM statements T=K?TM?@ are sufficient, but <?)TM?@
statement (2=<? is sufficient.
23.
When solve such ;ind of questions, we Hust need to ;now the ratio one price to another
price. )t is time waste to calculate one by one.
.oth two statements do not give the information, as well as their combination.
(nswer is ?
/". This question can be restated in several ways. 2et )or, , amount earned +i.e., amount
needed to purchase the Hac;et-. @ecall, )or, , 8ate * Ti&e. 'ince the number of
hours that either 0im or Tom need to wor; in order to purchase the Hac;et is given, we
need only ;now either personEs rate of pay to determine the cost of the Hac;etS hence,
the question can be restated as either PWhat is xQP or PWhat is yQP.
(lso, since the amount of time needed for either 0im or Tom to purchase the Hac;et is
given, it can be shown that the amount of time needed for them wor;ing together to
purchase the Hac;et can also be calculated. The formula )or, , 8ate * Ti&e also
applies when 0im and Tom wor; togetherS hence, only the combined rate of 0im and
Tom wor;ing together is required. 'ince the combined rate of two people wor;ing
together is equal to the sum of their individual rates, the question can also be restated
as PWhat is 9 + "QP
+1- )<'B::)A)?<T This statement gives only the relative earning power of 0im and
Tom. 'ince the original question states the amount of time needed for either 0im or
Tom to earn enough money to purchase the Hac;et, it also gives us the relative earning
power of 0im and Tom. Mence, statement +1- does not add any information to the
original question.
+/- 'B::)A)?<T 2et : , 1 Hac;et. 'ince Tom and 0im must " and 5 hours,
respectively, to earn enough to buy 1 Hac;et, in units of PHac;et per hour,P 0im wor;s at
the rate of 1D" Hac;ets per hour and Tom wor;s at the rate of 1D5 Hac;ets per hour.
Their combined rate is 1D" 4 1D5 , 5D/0 4 "D/0 , #D/0 Hac;ets per hour. 'ince Ti&e ,
)or,;8ate, Ti&e , 1 Hac;etD+#D/0 Hac;ets per hour- , /0D# hours.
'ince the combined pay rate of the 0im and Tom is equal to the sum of the individual pay
rates of the twoS hence, the combined pay rate in dollars per hour is 9 + ". When the two
wor; together, %&ount0arned , Co&3inedPay8ate * Ti&e , +9 + "- * #D/0. 'ince
statement +/- states that 9 + " , $"%.!5, this statement is sufficient to compute the cost of
the Hac;et +it is not necessary to ma;e the final calculation-.
The correct answer is ..
<ote )t is also not necessary to e*plicitly compute the time needed for 0im and Tom
wor;ing together to earn the Hac;et +/0D# hours-. )t is only necessary to recogniJe that this
number (an 3e (al(ulated in order to determine that +/- is sufficient.
/5. The question stem tells us that .ill has a stac; of $1, $5, and $10 bills in the ratio of
10 5 1 respectively. WeEre trying to find the number of $10 bills.
+1- )<'B::)A)?<T 'ince the ratio of the number of $1 bills to $10 bills is 10 1, the
dollar value of the $1 and $10 bills must be equal. Therefore statement +1- gives us no
new information, and we cannot find the number of $10 bills.
+/- 'B::)A)?<T The problem states that the number of $1, $5, and $10 bills is in the
ratio of
10 5 1, so letEs use an un;nown multiplier x to solve the problem.
Bsing x, we can see that there are 10x $1 bills with a value of $10x. :urthermore, there
are 5x $5 bills with a value of $/5x. :inally, there are 1x $10 bills with a value of $10x.
'tatement +/- says that the total amount he has is $//5, so we can set up an equation as
follows
$10x 4 $/5x 4 $10x , $//5
$"5x , $//5
x , 5
'ince there are 1x $10 bills this means that there are 5 $10 bills.
The correct answer is ..
/5. )t is tempting to view the information in the question as establishing a pattern as
follows
Kreen, >ellow, @ed, Kreen, >ellow, @ed, . . .
Mowever, consider that the following nonGpattern is also possible
Kreen, >ellow, @ed, Kreen, Kreen, Kreen, Kreen . . .
)<'B::)A)?<T This tells us that the 1&th tile is Kreen or @ed but this tells us
nothing about the /"th tile. 'tatement +1- alone is <=T sufficient.
)<'B::)A)?<T This tells us that the 1#th tile is >ellow or @ed but this tells us
nothing about the /"th tile. 'tatement +/- alone is <=T sufficient.
(<$ +/- )<'B::)A)?<T Together, the statements yield the following
possibilities for the 1&th and 1#th tiles
K>, K@, @>, or @@
Mowever, only K> adheres to the rules given in the question. Thus, we ;now that tile 1&
is green and tile 1# is yellow. Mowever, this does not help us to determine the color of
the ne*t tile, much less tile /" +the one as;ed in the question-. :or e*ample, the next tile
+tile /0- could be green or red. Thus, the statements ta;en together are still not sufficient.
The correct answer is ?.
/!. ?ach bas;et must contain at least one of each type of fruit. We also must ensure that
every bas;et contains less than twice as many apples as oranges. Therefore, the
minimum number of apples that we need is equal to the number of bas;ets, since we
can simply place one apple per bas;et +even if we had only 1 apple and 1 orange per
bas;et, we would not be violating any conditions-. )f we are to divide the /0 oranges
evenly, we ;now we will have 1, /, ", 5, 10, or /0 bas;ets +the factors of /0-. .ut
$1 bills $5 bills $10 bills Total
<umber 10 x 5 x 1 x 15 x
Ialue $10 x $/5 x $10 x $"5 x
because we donEt ;now the e*act number of bas;ets, we do not ;now how many
apples we need. Thus, the question can be rephrased as PMow many bas;ets are
thereQP
)<'B::)A)?<T This tells us only that the number of bas;ets is even +halving an
odd number of bas;ets would result in half of a bas;et-. 'ince we have /0 oranges
that must be distributed evenly among an even number of bas;ets, we ;now we
have /, ", 10, or /0 bas;ets. .ut because we still do not ;now e*actly how many
bas;ets we have, we cannot ;now how many apples we will need.
'B::)A)?<T This tells us that 10 oranges +half of the original /0- would not be
enough to place an orange in every bas;et. 'o we must have more than 10 bas;ets.
'ince we ;now the number of bas;ets is 1, /, ", 5, 10, or /0, we ;now that we
must have /0 bas;ets. Therefore, we ;now how many apples we will need.
The correct answer is ..
28.
2et the cost of each coat be *, the sales price be y. We Hust want to ;now what is /0+yG*-.
:or 1, we ;new that /0+/yG*-,/"00, insufficient to find yG*
:or /, we ;new that /0+y4/G*-,""0, we can get /0+yG*-,"00. )tEs sufficient.
(nswer is .
29.
:or 1, country ( can send # representatives, total number will #4&4!45454"1,!5N!5.
(nswer is ?
30.
2et number of rows is a, number of the chairs in a row is b.
'o, bGa,1
:rom 1, ab!/, a&, b#, sufficient alone.
:rom /, /bG1,1!, b,#, sufficient alone.
(nswer is $.
31.
'tatement 1 is obviously insufficient
'tatement /, let :riday be *. To obtain the least value of *, the other five days should
be, *G1, *G/, *G%, *G", *G5
'o, %&4*4*G14*G/4*G%4*G"4*G5,#0
5*,5!
*,5!D5N11
%/.
2et attend fee be *, number of person be y
:orm 1, +*G0.!5-+y4100-,*yGGGG100*G0.!5yG!5,0
:rom /, +*41.5-+yG100-,*y GGGG100*41.5yG150,0
Aombine 1 and /, we can get specific value of * and y.
(nswer is A
33.
Aombined 1 and /, three situations need to be studied
GGGG2ast wee;4this wee;O%5, then *,+510G"&0-D/,15, the number of the items is
"&0D15,%/
GGGG2ast wee;,%5, then *,"&0D%5,150D!. =r * can be resolved in the way
*,+510G"&0-D+14%D/-,1/, two result are conflict.
GGGG2ast wee;N,%5, then *,%0D+/8%D/-,10. The number of the items more than %5 ,+"&0G
%5810-D/0,5, so, total number is %545,"/
(bove all, answer is ?
34.
1- is sufficient.
/-. <o two members sold same number of tic;etsS the least numbers of the tic;ets they
sold would be 0, 1, /.
(nswer is $
35.
Pone ;ilogram of a certain coffee blend consists of V ;ilogram of type ) and > ;ilogram
of type ))P means that V4>,1
Aombined A,5.5V4&.5>, we get
V,+&.5GA-D/, >,+AG5.5-D/
Aombined AN,!.%, V,+&.5GA-D/O1./D/,0.5
(nswer is .
36.
)t is somewhat tric;y.
Bsually, we need two equations to solve two variables.
:or e*ample, in this question, from 1, *,y,5, from /, /1*4/%y,1%0, the answer should
be A.
(ctually, the variables in such questions should be integers. Thus, hopefully, we can solve
them with only one equation.
/1*4/%y,1%0, we try *,1, /, %, ",5..and find that only *,", y,/ can fulfill the
requirements. (nswer is ..
To sum up, please be careful when you met such questions.
37.
3ore than 10 Laperbac; boo;s, at least is 11, and cost at least $&&
:rom 1, 150D/5,5, at least 5 hardcover boo;s.
:rom /, /50G150G&&,//, is not enough to buy a hardcover boo;.
Aombined 1 and /, we ;now that 0uan bought 5 hardcover boo;s.
(nswer is A
38.
c,;*4t
)n last month, cost,1000;4tS profit,1000+;450- G+1000;4t- ,50000Gt, so, we need to
solve t.
:rom 1, 150000,10008+;450-, there is no information about t.
:rom /, +1000;4t-G+500;4t-,"5000, still cannot solve out t.
(nswer is ?
39.
+/45454"-D+500041/00041&000415000-850000 , /0
(nswer is .
40.
The fine for one day $0.1
The fine two days $0./, as it is less than $0.14$0.%
The fine for three days $0.", as it is less than $0./4$0.%
The fine for four days $0."4$0.%,$0.!, as it is less than $0."8/
(nswer is .
"1.
2et * be the height of the tree increase each year, then
7"45*G+"4"*-9D+"4"*- , 1D5
10* , "4 "*
*, /D%
"/.
)n the origin plan, each one should pay VDT.
(ctually, each of the remaining cowor;ers paid VD+TG'-.
Then, VD+TG'- G VDT , '8V D T+TG'-
"%. The business produced a total of "* ra;es from <ovember through :ebruary. The
storage situations were shown in the following table
'o, the total cost is 1"V80.1,1."V
3onth 3ar. (pr. 3ay 0une 0uly (ug. 'ept. =ct. Total
'torage !*D/ %* 5*D/ /* %*D/ * *D/ 0 1"*
44. In order to realize a profit, the company's revenue must be higher than the company's costs. We can
express this as an inequality using the information from the question:
If e distribute and move all terms to one side, e get:
We can factor this result:
When the value of p ma!es this inequality true, e !no e ill have a profit. When the value of p
does "#$ ma!e the inequality true, e ill not have a profit. When p equals % or 4, the product is
zero. &o the values of p that ill ma!e the inequality true 'i.e., ill yield a negative product( must be
either greater than 4, less than %, or beteen % and 4. $o determine hich is the case, e can test a
sample value from each interval.
If e try p ) *, e get:
&ince + is positive, e !no that values of p greater than 4 ill not ma!e the inequality true and thus
ill not yield a profit.
If e try p ) +, e get:
&ince + is positive, e !no that the values of p less than % ill not ma!e the inequality true and thus
ill not yield a profit.
If e try p ) %.*, e get:
&ince ,.+* is negative, e !no that values beteen % and 4 ill ma!e the inequality true and ill
thus yield a profit. &ince p can be any positive value less than -.. 'e cannot have a negative price
or a price of zero dollars(, there are -.. possible intervals beteen consecutive integer values of p.
$he interval % / p / 4 is 0ust one. $herefore, the probability that the company ill realize a profit is
-1-.. and the probability that it ill "#$ realize a profit is - , -1-.. or 221-...
$he correct anser is 3.
4*. $o calculate the average daily deposit, e need to divide the sum of all the deposits up to and
including the given date by the number of days that have elapsed so far in the month. 4or example, if
on 5une -% the sum of all deposits to that date is 6+.,+%., then the average daily deposit to that date
ould be .
We are told that on a randomly chosen day in 5une the sum of all deposits to that day is a prime
integer greater than -... We are then as!ed to find the probability that the average daily deposit up
to that day contains feer than * decimal places.
In order to anser this question, e need to consider ho the numerator 'the sum of all deposits,
hich is defined as a prime integer greater than -..( interacts ith the denominator 'a randomly
selected date in 5une, hich must therefore be some number beteen - and %.(.
4irst, are there certain denominators that 7 no matter the numerator 7 ill alays yield a quotient
that contains feer than * decimals8
9es. : fraction composed of any integer numerator and a denominator of - ill alays yield a quotient
that contains feer than * decimal places. $his ta!es care of 5une 1.
In addition, a fraction composed of any integer numerator and a denominator hose prime
factorization contains only +s and1or *s ill alays yield a quotient that contains feer than * decimal
places. $his ta!es cares of 5une 2, 4, 5, 8, 10, 16, 20, and 25.
Why does this or!8 ;onsider the chart belo:
3enominator
:ny Integer divided by this denominator ill yield either an integer quotient or
a quotient ending in:
< of 3ecimal
=laces
+ .* -
4 multiples of .+* maximum of +
* multiples of .+ -
> multiples of .-+* maximum of %
-. multiples of .- -
-? multiples of ..?+* maximum of 4
+. multiples of ..* maximum of +
+* multiples of ..4 maximum of +
What about the other dates in 5une8
If the chosen day is any other date, the denominator 'of the fraction that ma!es up the average daily deposit(
ill contain prime factors other than + and1or * 'such as % or @(. Aecall that the numerator 'of the fraction that
ma!es up the average daily deposit( is defined as a prime integer greater than -.. 'such as -.-(.
$hus, the denominator ill be composed of at least one prime factor 'other than + and1or *( that is not a
factor of the numerator. $herefore, hen the division ta!es place, it ill result in an infinite decimal. '$o
understand this principle in greater detail read the explanatory note that follos this solution.(
$herefore, of the %. days in 5une, only 2 '5une -, +, 4, *, >, -., -?, +., and +*( ill produce an average daily
deposit that contains feer than * decimal places: .
$he correct anser is 3.

Explanatory Note: Why ill an infinite decimal result henever a numerator is divided by a denominator
composed of prime factors 'other than + and1or *( that are not factors of the numerator8
;onsider division as a process that ends hen a remainder of . is reached.
Bet's loo! at - 'the numerator( divided by @ 'the denominator(, for example. If you divide - by @ on your
calculator, you ill see that it equals .-4+>... $his decimal ill go on infinitely because @ ill never divide
evenly into the remainder. $hat is, a remainder of . ill never be reached.
, and so on...
4or contrast, let's loo! at +% divided by *:
&o +%1* ) 4.?. When the first remainder is divided by *, the division ill end because the first remainder '%(
is treated as if it ere a multiple of -. to facilitate the division and * divides evenly into multiples of -..
Cy the same to!en, the remainder hen an odd number is divided by + is alays -, hich is treated as if it
ere -. to facilitate the division. -. divided by + is * 'hence the .*( ith no remainder.
When dividing by primes that are not factors of -. 'e.g., %, @, --, etc.(, hoever, the process continues
infinitely because the remainders ill alays be treated as if they ere multiples of -. but the primes cannot
divide cleanly into -., thus creating an endless series of remainders to be divided.
If the divisor contains +'s and1or *'s in addition to other prime factors, the infinite decimal created by the other
prime factors ill be divided by the +'s and1or *'s but ill still be infinite.
4?. It might be tempting to thin! that either statement is sufficient to anser this question. :fter all,
pouring ater from the larger container to the smaller container ill leave exactly + gallons of ater
in the larger container. Aepeating this operation tice ill yield 4 gallons of ater.
$he problem is , here ould these 4 gallons of ater accumulate8 We ill need to use one of the
containers. Doever, neither statement alone tells us hether one of the containers ill hold 4
gallons of ater.
#n the other hand, statements '-( and '+( ta!en together ensure that the first container can hold at
least 4 gallons of ater. We !no this because 'from statement -( the first container holds + gallons
more than the second container, hich 'from statement +( holds + gallons more than the third
container, hich must have a capacity greater than ..
&ince e !no that the first container has a capacity of at least 4 gallons, there are several ays of
measuring out this exact amount.
#ne method is as follos: ;ompletely fill the first container ith ater. $hen pour out 0ust enough
ater from the first container to fill the third container to the brim. "o, 4 gallons of ater remain in
the first container.
:lternatively: 4ill the first container to the brim. =our out 0ust enough ater from the first container
to fill the second container to the brim. $here are no + gallons of ater in the first container. "o
pour ater from the second container to fill the third container to the brim. $here are no + gallons
of ater in the second container. 4inally, pour all the ater from the second container into the first
container. $here are no 4 gallons of ater in the first container.
4@. We can anser this by !eeping trac! of ho many cubes are lopped off of each side as the cube is
trimmed '-. x -. E -. x 2 E 2 x 2 E ...(, but this approach is tedious and error prone. : more
efficient method is to determine the final dimensions of the trimmed cube, then find the difference
beteen the dimensions of the trimmed and original cubes.
Bet's call the first face :, second face C, and third face ;. Cy the end of the operation, e ill have
removed + layers each from faces C and ;, and % layers from face :. &o C no is > cubes long, ; is >
cubes long, and : is @ cubes long. $he resulting solid has dimensions > x > x @ cubes or 44> cubes.
We began ith -... cubes, so -... , 44> ) **+. $hus, **+ cubes have been removed.
$he correct anser is C.
4>. In order to determine the length of the line, e need to !no ho many people are standing in it. $hus,
rephrase the question as follos: Do many people are standing in the line8 &tatement '-( says that there
are three people in front of ;handra and three people behind Fen. ;onsider the folloing different scenarios:
$he line might loo! li!e this: 'Cac!( G G G Fen G ;handra G G G '4ront(
#A $he line might loo!s li!e this: 'Cac!( ;handra G G Fen '4ront(
$he number of people in the line depends on several factors, including hether ;handra is in front of Fen
and ho many people are standing beteen ;handra and Fen. &ince there are many different scenarios,
statement '-( is not sufficient to anser the question.
&tatement '+( says that to people are standing beteen ;handra and Fen. Dere, e don't !no ho many
people are ahead or behind Fen and ;handra. &ince there are many different scenarios, statement '+( is not
sufficient to anser the question.
$a!ing both statements together, e still don't !no hether ;handra is in front of Fen or vice versa, and
therefore e still have to different possibilities:
$he line might loo! li!e this: 'Cac!( G G G Fen G G ;handra G G G '4ront(
#A $he line might loo! li!e this: 'Cac!( ;handra G G Fen '4ront(
$herefore, the correct anser is 'H(: &tatements '-( and '+( $#IH$DHA are "#$ sufficient.
42. Cegin by rephrasing, or simplifying, the original question. &ince the rules of the game involve the
negative of the sum of to dice, one ay of restating this problem is that hoever gets the higher
sum B#&H& the game. $hin!ing about the sum of the to dice is easier than thin!ing about the
negative of the sum of the to dice. $hus, let's rephrase the question as: Who lost the game8
'Fnoing this ill obviously allo us to anser the original question, ho won the game.(
&tatement '-( gives us information about the first of "ina's dice, but it does not tell us anything about the
second. ;onsider the folloing to possibilities:
"ina's 4irst Aoll "ina's &econd Aoll $eri's &um Digher &um ) Boser
;:&H #"H 3 * 1 "ina
;:&H $W# 3 7* 1 $eri
"otice that in both cases, "ina's first roll is greater than $eri's &um. Doever, in ;ase #ne "ina loses, but in
;ase $o $eri loses. $hus, this information is not sufficient to anser the question.
&tatement '+( gives us information about the second of "ina's dice, but it does not tell us anything about the
first. Jsing the same logic as for the previous statement, this is not sufficient on its on to anser the
question.
;ombining the information contained in both statements, one may be tempted to conclude that "ina's sum
must be higher than $eri's sum. Doever, one must test scenarios involving both positive and negative rolls.
;onsider the folloing to possibilities.
"ina's 4irst Aoll "ina's &econd Aoll $eri's &um Digher &um ) Boser
;:&H #"H 3 4 5 "ina
;:&H $W# 3 4 5 $eri
"otice that in both cases, "ina's first roll is greater than $eri's &um and "ina's second roll is greater than
$eri's sum. Doever, in ;ase #ne "ina loses, but in ;ase $o $eri loses. $hus, this information is not
sufficient to anser the question.
$he correct anser is H: &tatements '-( and '+( $#IH$DHA are "#$ sufficient.
*.. Hvery third :lb gives a clic!. $his means no clic! is aarded until the third :lb is captured. $he
second clic! is not aarded until the sixth :lb is captured. &imilarly, a tic! is not aarded until the
fourth Cer! is captured.
We are told that the product clic!s x tic!s ) @@. $hus, there are four possibilities: - K @@, @ K --, -- K @, @@
K -.
;lic!s :arded :lbs ;aptured $ic!s :arded Cer!s ;aptured
- %, 4 or * @@ %.>, %.2, %-., or %--
@ +-, ++, or +% -- 44, 4*, 4? or 4@
-- %%, %4, or %* @ +>, +2, %. or %-
@@ +%-, +%+ or +%% - 4, *, ?, or @
&tatement '-( tells us that the difference beteen :lbs captured and Cer!s captured is @. Boo!ing at the
chart, the only ay to get a difference of @ beteen :lbs captured and Cer!s captured is ith %* :lbs and +>
Cer!s. $herefore, statement '-( is sufficient to anser the question,,there must have been %* :lbs captured.
&tatement '+( says the number of :lbs captured is divisible by 4. :gain, loo!ing at the chart, e see that the
number of :lbs captured must be 4 or +%+. $herefore, statement '+( is not sufficient to anser the
question,,e do not !no ho many :lbs ere captured.
$he correct anser is :: &tatement '-( alone is sufficient, but statement '+( alone is not sufficient.
51.
The question gives a function with two unknown constants and two data points. In order
to solve for the position of the object after 4 seconds, we need to frst solve for the
contants r and b. e can do this b! creating two equations fro" the two data points
given#
p$%& ' 41 ' r$%& ( 5$%&
%
) b
41 ' %r ( %* ) b
+1 ' %r ) b
p$5& ' %+ ' r$5& ( 5$5&
%
) b
%+ ' 5r ( 1%5 ) b
151 ' 5r ) b
e can now solve these equations for r and b using substitution#
+1 ' %r ) b
$+1 ( %r& ' b
151 ' 5r ) b
151 ' 5r ) $+1 ( %r&
151 ' ,r ) +1
-* ' ,r
r ' ,*
.ubstituting back in, we can fnd b#
+1 ' %r ) b
+1 ' %$,*& ) b
b ' 1
.o, we can rewrite the original function and plug in t ' 4 to fnd our answer#
p$t& ' ,*t ( 5t
%
) 1
p$4& ' ,*$4& ( 5$4&
%
) 1
p$4& ' 1%* ( /* ) 1
p$4& ' 41
The correct answer is 0.
5%.
1et us call the Trussian2s current age a. Therefore the Trussian2s current weight is .
.eventeen !ears after he is twice as old as he is now, the Trussian2s age will be
and his weight will therefore be .e are told that the Trussian2s current weight,
, is three keils less than his future weight, . Therefore, .e
can solve the equation as follows#

a ' 1+ or 4. 3owever, we are told that the Trussian is a teenager so he "ust be 1+
!ears old.
The correct answer is 4.
5,.
This proble" is easier to think about with real values.
1et2s assu"e that there are % high level o5cials. This "eans that each of these % high
level o5cials supervises 4 $or x
%
& "id6level o5cials, and that each of these 4 "id6level
o5cials supervises / $or x
,
& low6level o5cials.
It is possible that the supervisors do not share an! subordinates. If this is the case, then,
given % high level o5cials, there "ust be %$4& ' / "id6level o5cials, and /$/& ' +4 low6
level o5cials.
7lternativel!, it is possible that the supervisors share all or so"e subordinates. In other
words, given % high level o5cials, it is possible that there are as few as 4 "id6level
o5cials $as each of the % high6level o5cials supervise the sa"e 4 "id6level o5cials& and
as few as / low6level o5cials $as each of the 4 "id6level o5cials supervise the sa"e /
low6level o5cials&.
.tate"ent $1& tells us that there are fewer than +* low6level o5cials. This alone does not
allow us to deter"ine how "an! high6level o5cials there are. 8or e9a"ple, there "ight
be % high level o5cials, who each supervise the sa"e 4 "id6level o5cials, who, in turn,
each supervise the sa"e / low6level o5cials. 7lternativel!, there "ight be , high6level
o5cials, who each supervise the sa"e - "id6level o5cials, who, in turn, each supervise
the sa"e %: low6level o5cials.
.tate"ent $%& tells us that no o5cial is supervised b! "ore than one person, which
"eans that supervisors do not share an! subordinates. 7lone, this does not tell us
an!thing about the nu"ber of high6level o5cials.
4o"bining state"ents 1 and %, we can test out di;erent possibilities.
If x ' 1, there is 1 high6level o5cial, who supervises 1 "id6level o5cial $1
%
' 1&, who, in
turn, supervises 1 low6level o5cial $1
,
' 1&.
If x ' %, there are % high6level o5cials, who each supervise a unique group of 4 "id6
level o5cials, !ielding / "id6level o5cials in total. <ach of these / "id6level o5cials
supervise a unique group of / low6level o5cials, !ielding +4 low6level o5cials in total.
3owever, this cannot be the case since we are told that there are fewer than +* low6
level o5cials.
Therefore, based on both state"ents taken together, there "ust be onl! 1 high6level
o5cial. The correct answer is 4# =>T3 state"ents T>?<T3<@ are su5cient, but
A<IT3<@ state"ent 71>A< is su5cient.
54.
Bse algebra to solve this proble" as follows#
1et the x ' the nu"ber of donuts Ci" originall! ordered. .ince he paid D15 for these
donuts, the price per donut for his original order is D15E9.
hen he leaves, Ci" receives , free donuts changing the price per donut to D15E$9 ) ,&.
In addition, we know that the price per doFen donuts was D% per doFen cheaper when he
leaves, equivalent to a per donut savings of D%E1% ' 1E+ dollars.
Bsing this infor"ation, we can set up an equation that states that the original price per
donut less 1E+ of a dollar is equal to the price per donut after the addition of , donuts#
e can now solve for x as follows#
The onl! positive solution of 9 is 15. 3ence, Ci" left the donut shop with 9 ) , ' 1/
donuts. The correct answer is 7.
55.
In questions like this, it helps to record the given infor"ation in a table. Bpon initial
reading, the second sentence is probabl! ver! confusing but what is clear is that it
discusses the ages of the two bo!s at two di;erent points in ti"e# let2s refer to the" as
GnowH, and GthenH. .o, let2s construct a table such as the one below. 1et x and y denote
the bo!s2 ages GnowH#
Cohnn!2s age=obb!2s age
Aowx y
then
Aow, re6read the frst few words of the second sentence# GCohnn!2s age now is the sa"e
as =obb!2s age . . . IthenJH. e can fll in one "ore entr! of the table as shown#
Cohnn!2s age=obb!2s age
Aowx y
then x
8inall!, the rest of the second sentence tells us that GthenH was the ti"e when Cohnn!2s
age was half =obb!2s current ageK i.e., Cohnn!2s age GthenH was $1E%&y. e can co"plete
the table as follows#
Cohnn!2s age=obb!2s age
Aowx y
then$1E%&y x
>ne wa! to solve this proble" is to realiFe that, as two people age, the ratio of their ages
changes but the di;erence in their ages re"ains constant. In particular, the di;erence in
the bo!s ages Gnow2H "ust be the sa"e as the di;erence in their ages GthenH. This leads
to the equation# y 6 x ' x 6 $1E%&y, which reduces to x ' $,E4&yK Cohnn! is currentl! three6
fourths as old as =obb!.
ithout another equation, however, we can2t solve for the values of either x or y.
$7lternativel!, we could co"pute the elapsed ti"e between GthenH and GnowH for each
bo! and set the two equalK this leads to the sa"e equation as above.&
$1& IA.B88I4I<AT# =obb!2s age at the ti"e of Cohnn!2s birth is the sa"e as the di;erence
between their ages, y 6 x. .o state"ent $1& tells us that y ' 4$y 6 x&, which reduces to x
' $,E4&y. This adds no "ore infor"ation to what we alread! knewL .tate"ent $1& is
insu5cient.
$%& .B88I4I<AT# This tells us that =obb! is + !ears older than Cohnn!K i.e., y ' x ) +. This
gives us a second equations in the two unknowns so, e9cept in so"e rare cases, we
should be able to solve for both 9 and ! 66 state"ent $%& is su5cient. Cust to verif!,
substitute x ' $,E4&y into the second equation to obtain y ' $,E4&y ) + , which i"plies y
' %4. =obb! is currentl! %4 and Cohnn! is currentl! 1/.
The correct answer is =, .tate"ent $%& 71>A< is su5cient to answer the question, but
state"ent $1& alone is not.
56.
8irst, let c be the nu"ber of cash"ere blaFers produced in an! given week and let m be
the nu"ber of "ohair blaFers produced in an! given week. 1et p be the total proft on
blaFers for an! given week. .ince the proft on cash"ere blaFers is D4* per blaFer and
the proft on "ohair blaFers is D,5 per blaFer, we can construct the equation p = 4*c +
,5m. In order to know the "a9i"u" potential value of p, we need to know the
"a9i"u" values of c and m.
.tate"ent $1& tells us that the "a9i"u" nu"ber of cutting hours per week is %** and
that the "a9i"u" nu"ber of sewing hours per week is %**.
.ince it takes 4 hours of cutting to produce a cash"ere blaFer and 4 hours of cutting to
produce a "ohair blaFer, we can construct the following inequalit!# .
.ince it takes + hours of sewing to produce a cash"ere blaFer and % hours of sewing to
produce a "ohair blaFer, we can construct the following inequalit!# .
In order to "a9i"iFe the nu"ber of blaFers produced, the co"pan! should use all
available cutting and sewing ti"e. .o we can construct the following equations#
.ince both equations equal %**, we can set the" equal to each other and solve#
.o when m ' %5 and c = %5, all available cutting and sewing ti"e will be used. If p =
4*c ) ,5m, the proft in this scenario will be 4*$%5& ) ,5$%5& or D1,/:5. Is this the
"a9i"u" potential proftM
.ince the proft "argin on cash"ere is higher, "ight it be possible that producing onl!
cash"ere blaFers would be "ore proftable than producing both t!pesM If no "ohair
blaFers are "ade, then the largest nu"ber of cash"ere blaFers that could be "ade will
be the value of c that satisfes +c ' %** $re"e"ber, it takes + hours of sewing to "ake a
cash"ere blaFer&. .o c could have a "a9i"u" value of ,, $the co"pan! cannot sell 1E,
of a blaFer&. .o producing onl! cash"ere blaFers would net a potential proft of 4*$,,& or
D1,,%*. This is less than D1,/:5, so it would not "a9i"iFe proft.
.ince "ohair blaFers take less ti"e to produce, perhaps producing onl! "ohair blaFers
would !ield a higher proft. If no cash"ere blaFers are produced, then the largest
nu"ber of "ohair blaFers that could be "ade will be the value of m that satisfes 4m '
%** $re"e"ber, it takes 4 hours of cutting to produce a "ohair blaFer&. .o m would
have a "a9i"u" value of 5* in this scenario and the proft would be ,5$5*& or D1,:5*.
This is less than D1,/:5, so it would not "a9i"iFe proft.
.o producing onl! one t!pe of blaFer will not "a9i"iFe potential proft, and producing
both t!pes of blaFer "a9i"iFes potential proft when m and c both equal %5.
.tate"ent $1& is su5cient.
.tate"ent $%& tells us that the wholesale cost of cash"ere cloth is twice that of "ohair
cloth. This infor"ation is irrelevant because the cost of the "aterials is alread! taken
into account b! the proft "argins of D4* and D,5 given in the question ste".
.tate"ent $%& is insu5cient.
The answer is 7# .tate"ent $1& alone is su5cient, but state"ent $%& alone is not.
57.
<ach !ear, the age of the bo! increases b! 1. <ach !ear, the su" of the ages of the two
girls increases b! % $as each girl gets older b! one !ear, and there are two of the"&.
1et2s sa! that the age of the bo! toda! is equal to x, while the co"bined ages of the girls
toda! is equal to y.
Then, ne9t !ear the fgures will be x ) 1 and y ) %, respectivel!. The proble" states that
these two fgures will be equal, which !ields the following equation#
x ) 1 ' y ) % which can be si"plifed to x = y + 1
$This is consistent with the fact that the su" of the ages of the two girls toda! is s"aller
than the age of the bo! toda!.&
Three !ears fro" now, the co"bined age of the girls will be y ) ,$%& ' y ) +. Three !ears
fro" now, the bo!2s age will be x ) ,. Bsing the fact $fro" above& that x ' y ) 1, the
bo!2s age three !ears fro" now can be written as x ) , ' $y ) 1& ) , ' y ) 4.
The proble" asks for the di;erence between the age of the bo! three !ears fro" toda!
and the co"bined ages of the girls three !ears fro" toda!. This di;erence equals y ) 4 (
$y ) +& ' (%.
The correct answer is 0.
Nlug in real nu"bers to see if this "akes sense.
1et the girls be 4 and + in age. The su" of their ages toda! is 1*. The bo!2s age toda! is
then $1* ) 1& ' 11. Three !ears fro" toda!, the girls will be : and - respectivel!, so their
co"bined age will be 1+. Three !ears fro" toda!, the bo! will be 14.
=e careful# The question asks for the di;erence between the bo!2s age and the su" of
the girls ages three !ears fro" toda!. hich one will be !oungerM The bo!. .o the
di;erence between the bo!2s age and the co"bined age of the girls will be a negative
value# 14 ( 1+ ' ( %.
58.
Bse a chart to keep track of the ages in this proble"#
x !ears ago A> in x !ears
4ore! C 6 x C C ) x
Tania T 6 x T T ) x
Then write algebraic e9pressions to represent the infor"ation given in the proble"#
x !ears ago, 4or! was one ffth as old as Tania
5$C ( x& ' T ( x
5C ( 5x ' T ( x
5C ( T ' 4x
In 9 !ears, Tania will be twice as old as 4or!
%$C ) x& ' T ) x
%C ) %x ' T ) x
T ( %C ' x
.ubstitute T 6 %C in for x in the frst equation and solve#
5C 6 T ' 4$T 6 %C&
5C 6 T ' 4T 6 /C
1,C ' 5T
The correct answer is 4.
5-.
The question does not ask for the actual nu"ber of !ears ago that ani"al z beca"e
e9tinct. Instead it asks for t, the nu"ber of !ears scientists predicted it would take for
ani"al z to beco"e e9tinct.
$1& IA.B88I4I<AT# This tells us that ani"al z beca"e e9tinct 4 !ears ago but it does not
provide infor"ation about t.
$%& IA.B88I4I<AT# This provides a relationship between the predicted ti"e of e9tinction
ti"e and the actual ti"e of e9tinction but does not provide an! actual values for either.
$1& 7A0 $%& IA.B88I4I<AT# The easiest wa! to approach this proble" is to i"agine a ti"e
line fro" * to 1*. The scientists "ade their prediction 1* !ears ago, or at * !ears.
8ro" state"ent $1& we know that ani"al z beca"e e9tinct 4 !ears ago, or at + !ears.
8ro" state"ent $%& we know that if the scientists had e9tended their prediction b! ,
!ears the! would have been incorrect b! % !ears. The ke! to this question is to
realiFe that Oincorrect b! % !earsO could "ean % !ears in either direction# + ) % ' / !ears
or + ( % ' 4 !ears.
8ro" here, we can write two si"ple equations#
t ) , ' / >@ t ) , ' 4
t ' 5 t ' 1
This gives us two di;erent values for t, which "eans that $1& and $%& together are not
su5cient to co"e up with one defnitive value for t. The correct answer is <.
50.
To answer this question, we need to minimiJe the value of l , +!.5 C x-
"
4 &.#!
1.05
. 'ince we
do not need to determine the actual minimum longevity, we do not need to find the
value of the second component in our formula, &.#!
1.05
, which will remain constant for
any level of x. Therefore, to minimiJe longevity, we need to minimiJe the value of the
first component in our formula,
i.e. +!.5 C x-
"
. 'ince we are raising the e*pression +!.5 C x- to an even e*ponent, ", the
value of
+!.5 C x-
"
will always be nonGnegative, i.e. positive or Jero. Thus, to minimiJe this
outcome, we need to find the value of x for which +!.5 C x-
"
, 0.
+!.5 C x-
"
, 0
!.5 C x , 0
x , !.5
Therefore, the metal construction will have minimal longevity for the value of x , !.5,
i.e. when the density of the underlying material will be equal to !.5 gDcm%.
The correct answer is A.
Calculations, Exponents, Basic Algebra
1.
The frst culprit in this e9pression is the radical in the deno"inator. @adicals in the
deno"inator are dealt with b! "ultipl!ing the fraction with an e9pression that is equal to
1 but contains a Gcanceling radicalH in both the nu"erator and deno"inator.
8or e9a"ple, is si"plifed b! "ultipl!ing b! .
In the case of a co"ple9 radical, such as , we "ultipl! b! the conjugate,
as follows#
.i"plif!ing radicals in the deno"inator with conjugate radical e9pressions is ver!
useful on challenging ?P7T radical questions.
The correct answer is 4.
2.
The ?P7T does not require !ou to know how to evaluate an integral root of an! general
integer, but !ou are e9pected to understand how to evaluate an integer raised to an
integer power. 3ence, !ou should i""ediatel! realiFe that there "ust be a wa! we can
transfor" each of the e9pressions into an e9pression that we can evaluate.
The "ost obvious wa! to transfor" a root into a power is to raise it to a higher power.
.ince we are tr!ing to co"pare the e9pressions, a reasonable transfor"ation is to raise
each of the e9pressions to the sa"e power. hat power should we useM
.ince , in order to get integral powers of x $i.e., nEm is an integer&
we should raise all of the roots to the 1east 4o""on Pultiple of the ms. e have roots
of ,, +, 1*, and 15, so the 14P is equal to ,*. Therefore, we should raise each of the
e9pressions to the ,*
th
power as follows#
Thus, the original e9pressions in increasing order are as follows#
.
3.
In order to rid the e9pression of square roots, let2s frst square the entire e9pression. e
are allowed to do this as long as we re"e"ber to OunsquareO whatever solution we get at
that end.
Aotice that the new e9pression is of the for" where
.
@ecall that . This is one of the ?P7T2s favorite e9pressions.
@eturning to our e9pression#
, while and .
Aotice that neatl! si"plifes to 4/. This leaves onl! the e9pression left to
si"plif!.
In order to si"plif! , recall that .
Thus, .
Aotice that the e9pression under the square root sign is of the for" . 7nd
recall that . This is another one of the ?P7T2s favorite
e9pressions.
@eturning to our e9pression#
.
8inall! then# .
=ut now we "ust re"e"ber to OunsquareO $or take the square root of& our answer#
.
Therefore, the correct answer is 0.
4. e can si"plif! the question as follows#
/
a
$1E4&
b
' M Q=reak all non6pri"es down to pri"es.R
$%
,
&
a
$%
6%
&
b
' M QPultipl! e9ponents taken on the sa"e base.R
$%
,a
&$%
6%b
& ' M Q7dd e9ponents since the two bases are equal.R
%
,a 6 %b
' M
e can rephrase the question as Owhat is ,a 6%bMO
$1& .B88I4I<AT# b ' 1.5a, so %b ' ,a. This "eans that ,a 6 %b ' *.
$%& IA.B88I4I<AT# This state"ent gives us no infor"ation about b.
The correct answer is 7
5.
The distance fro" G to H is 5
1,
6 5
1%
.
The distance between and two consecutive points is constant, so the distance fro" 7 to
? will be + ti"es the distance fro" G to H or +$5
1,
( 5
1%
&.
The value of A, therefore, will be equal to the value of G "inus the distance fro" A to G#
5
1%
( +$5
1,
( 5
1%
& 5
1%
( +Q5
1%
$5 ( 1&R 5
1%
( +$5
1%
&$4&
5
1%
$1 ( %4& $6%,&5
1%
.
The correct answer is =.
+. $,
5x
) ,
5x
+ ,
5x
&$4
5x
+ 4
5x
) 4
5x
) 4
5x
& '
,
5x
$1 ) 1 ) 1& S 4
5x
$1 ) 1 ) 1 ) 1& '
,$,
5x
& S 4$4
5x
& '
,
5x)1
S

4
5x)1
'
$, S 4&
5x)1
'
1%
5x)1
@e"e"ber that when !ou "ultipl! di;erent bases raised to the .7P< e9ponent, the
product is si"pl! the product of the bases raised to their co""on e9ponent.
The correct answer is 7.
:.
8irst, let us si"plif! the e9ponential equation#
$%
a
&$,
b
&$5
c
& ' 1%$%
k
&$,
l
&$5
m
&
$%
a
&$,
b
&$5
c
& ' $,&$4&$%
k
&$,
l
&$5
m
&
$%
a
&$,
b
&$5
c
& ' $,
1
&$%
%
&$%
k
&$,
l
&$5
m
&
$%
a
&$,
b
&$5
c
& ' $%
k+2
&$,
l ) 1
&$5
m
&
hen the bases on both sides of an equation are equal and the bases are pri"e
nu"bers, the e9ponents of the respective bases "ust also be equal# a ' k ) %K b ' l ) 1K
and c ' m. Aow recall that a, b, and c represent the hundreds, tens, and units digits of
the three6digit integer xK si"ilarl!, k, l, and m represent the hundreds, tens, and units
digits of the three6digit integer y.
Therefore, the hundreds digit of x is % greater than the hundreds digit of yK the tens digit
of x is 1 greater than the tens digit of yK fnall!, the units digit of x is equal to the units
digit of y. Bsing this infor"ation, we can set up our subtraction proble" and fnd the
value of $x ( y&#
abc
klm
%1*
The correct answer is 4.
/. $1& .B88I4I<AT# .tate"ent$1& tells us that x T %
,4
, so we want to prove that %
,4
T
1*
1*
. e2ll prove this b! "anipulating the e9pression %
,4
.
%
,4
' $%
4
&$%
,*
&
%
,4
' 1+$%
1*
&
,
Aow %
1*
' 1*%4, and 1*%4 is greater than 1*
,
. Therefore#
%
,4
T 1+$1*
,
&
,
%
,4
T 1+$1*
-
&
%
,4
T 1.+$1*
1*
&.
.ince %
,4
T 1.+$1*
1*
& and 1.+$1*
1*
& T 1*
1*
, then %
,4
T 1*
1*
.
$%& .B88I4I<AT# .tate"ent $%& tells us that that x ' %
,5
, so we need to deter"ine if %
,5
T
1*
1*
. .tate"ent $1& showed that %
,4
T 1*
1*
, therefore %
,5
T 1*
1*
.
The correct answer is 0.
-. 7 radical e9pression in a deno"inator is considered non6standard. To eli"inate a
radical in the deno"inator, we can "ultipl! both the nu"erator and the
deno"inator b! the conjugate of that deno"inator.
The correct answer is 4
1*. 8irst rewrite the e9pression in the question using onl! pri"e bases $4 is not
pri"e&, as follows# %
a
%
%b
.

$1& .B88I4I<AT# e can substitute 6%b for a into the e9pression in the question. hat is
the value of $%
6%b
&$%
%b
&M

This can be si"plifed to %
6%b)%b
' %
*
' 1.

$%& IA.B88I4I<AT# e have no infor"ation about the value of a.

The correct answer is 7.
11. 7n e;ective strateg! for proble"s involving e9ponents is to break the bases of all
the e9ponents into pri"e factors. This technique will allow us to co"bine like
ter"s#
%:
4x ) %
S 1+%
6%x
S ,+
x
S -
+ ( %x
' 1
$,
,
&
4x ) %
S $% S ,
4
&
6%x
S $%
%
S ,
%
&
x
S $,
%
&
+ ( %x
' 1
,
1%x ) +
S %
6%x
S ,
6/x
S %
%x
S ,
%x
S ,
1% ( 4x
' 1
%
6%x ) %x
S ,
1%x ) + ( /x ) %x ) 1% ( 4x
' 1
%
*
S ,
%x ) 1/
' 1
,
%x ) 1/
' 1
,
%x ) 1/
' ,
*
%x ) 1/ ' *
%x ' 61/
x ' 6-
The correct answer is 7.
1%.
1et2s rewrite the right side of the equation in base % and base ,# $%
%x)1
&$,
%y61
& '
$%
,
&
x
$,
,
&
y
.

This can be rewritten as#

$%
%x)1
&$,
%y61
& ' %
,x
,
,y
.ince both bases on either side of the equation are pri"e, we can set the e9ponents of
each respective base equal to one another#
%x ) 1 ' ,x, so x ' 1
%y ( 1 ' ,y, so y ' 61
Therefore, x ) y ' 1 ) $61& ' *.
The correct answer is 4
1,.
=efore dealing with this e9pression, it is helpful to re"e"ber several general e9ponent
rules#
hen "ultipl!ing e9pressions with the sa"e base, 700 the e9ponents frst#
$,
%
&$,
,
& ' $,&$,&$,&$,&$,& ' ,
5
' ,
% ) ,
hen dividing e9pressions with the sa"e base, .B=T@74T the e9ponents frst#
$,
5
&E$,
%
& ' $,&$,&$,&$,&$,& E $,&$,& ' $,&$,&$,& ' ,
,
' ,
5 6 %
hen raising a power to a power, co"bine e9ponents b! PB1TIN1UIA? the"#
$,
%
&
4
' $,
%
&$,
%
&$,
%
&$,
%
& ' $, S ,&$, S ,&$, S ,&$, S ,& ' $,&$,&$,&$,&$,&$,&$,&$,& ' ,
/
' ,
%$4&
In this question, we are asked to solve an e9pression with "an! e9ponents, but none of
the" have co""on bases, at least not as currentl! written. 3owever, so"e of the bases
have pri"e factors in co""on. 1ook at each part of the e9pression and break each into
its factored for".
If we look at the ter"s in the nu"erator on the left side#
+
%
' $% S ,&
%
' %
%
S ,
%

44 ' % S % S 11 ' %
%
S 11
1

5
x
cannot be factored
%* ' % S % S 5 ' %
%
S 5
1
Aote that several of the ter"s in the nu"erator have bases in co""on, so the
nu"erator si"plifes b! 700IA? the e9ponents of those ter"s#
$%
%
S ,
%
&$%
%
S 11
1
&$5
x
&$%
%
S 5
1
& ' $%
+
&$,
%
&$5
$x ) 1&
&$11
1
&
If we look at the ter"s in the deno"inator on the left side#
/
%
' $% S % S %&
%
' $%
,
&
%
' %
+

- ' , S , ' ,
%

>n the right side of the equation#
1,:5 ' 5 S 5 S 5 S 11 ' 5
,
S 11
1
Aow that each e9ponent and large nu"ber is e9pressed in ter"s of its pri"e factors, we
can put the equation back together. Then we2ll see what can cancel to si"plif! the entire
equation#
$%
+
&$,
%
&$5
$x ) 1&
&$11
1
& V $%
+
&$,
%
& ' 5
,
S 11
1
4anceling the %
+
and ,
%
that appear in the nu"erator and deno"inator of the left side,
and canceling the 11 that appears on each side of the equal sign#
5
$x ) 1&
' 5
,
x ) 1 ' ,
x ' %
The correct answer is 0.
14.
If we know y, we can solve for x. Thus, we can rephrase the question, Ohat is yMO
$1& .B88I4I<AT# If y
%
' +%5, we know that y ' %5 or 6%5. 3owever, since 5
x
will be
positive no "atter what x is, and 5
x
' y, then y "ust be positive. Thus, y "ust be %5. If
y is %5, we know that x ' %.
$%& .B88I4I<AT# If y
,
' 15,+%5, y "ust equal %5. $Tip# In order to understand a nu"ber
like 15,+%5 e9ponentiall!, tr! breaking it down into its roots. 15,+%5 ' 5 S 5 S 5 S 5 S 5
S 5 ' %5 S %5 S %5 ' %5
,
.& If y is %5, we know that x ' %.
The correct answer is 0.
15.
This question "ight be rephrased G3ow "an! golf balls do end! and Nedro have
co"binedMH >therwise, we "ust si"pl! fnd the nu"ber of balls possessed b! Ci".
$1& IA.B88I4I<AT# >bserve that Ci" could have % balls and end! +, or Ci" could have ,
balls while end! has -.
$%& IA.B88I4I<AT# If Nedro has 1E% of the golf balls, Nedro has 1% balls. 3owever, this
state"ent gives no infor"ation about the nu"ber of balls possessed b! Ci" or end!.
$1& 7A0 $%& .B88I4I<AT# .tate"ent $%& tells us that Nedro has 1% balls. Therefore, end!
and Ci" collectivel! have the re"aining 1% balls. .tate"ent $1& tells us that Ci" has 1E, of
the nu"ber of end!Js golf balls. 1et j ' the nu"ber of Ci"Js golf balls and w ' the
nu"ber of end!Js golf balls.
j ' wE,
Pultipl!ing both sides b! , !ields
,j ' w
?iven that j ) w ' 1%, we can substitute that j ) ,j ' 1%. If 4j ' 1%, j ' ,.
The correct answer is 4.
1+.
The best wa! to answer this question is to use the e9ponential rules to si"plif!
the question ste", then anal!Fe each state"ent based on the si"plifed equation.
$,
%:
&$,5
1*
&$z& ' $5
/
&$:
1*
&$-
14
&$x
y
& =reak up the ,5
1*
and si"plif! the -
14
$,
%:
&$5
1*
&$:
1*
&$z& ' $5
/
&$:
1*
&$,
%/
&$x
y
& 0ivide both sides b! co""on ter"s 5
/
, :
1*
, ,
%:
$5
%
&$z& ' ,x
y
$1& .B88I4I<AT# 7nal!Fing the si"plifed equation above, we can conclude that z "ust
have a factor of , to balance the , on the right side of the equation. .tate"ent $1& sa!s
that z is pri"e, so z cannot have another factor besides the ,. Therefore z ' ,.
.ince z ' ,, the left side of the equation is :5, so x
y
' %5. The onl! integers greater than
1 that satisf! this equation are x ' 5 and y ' %, so state"ent $1& is su5cient. Nut
di;erentl!, the e9pression x
y
"ust provide the two fves that we have on the left side of
the equation. The onl! wa! to get two fves if x and y are integers greater than 1 is if x '
5 and y ' %.
$%& .B88I4I<AT# 7nal!Fing the si"plifed equation above, we can conclude that x "ust
have a factor of 5 to balance out the 5
%
on the left side. .ince state"ent $%& sa!s that x is
pri"e, x cannot have an! other factors, so x ' 5. Therefore state"ent $%& is su5cient.
The correct answer is 0.
1:. It is te"pting to e9press both sides of the equation 4
4x
' 1+** as powers of 4 and
to tr! and solve for x. 3owever, if we do that, we get a power of fve on the right
side as well#
4
4x
' 1+ S 1**
4
4x
' 4
%
S 4 S %5
4
4x
' 4
,
S 5
%

It beco"es clear that x is not an integer and that we canJt solve the question this
wa!.
1etJs tr! "anipulating the e9pression about which we are being asked.
$4
x1
&
%
' 4
%x %
If we further si"plif! we get the e9pression 4
%x
E4
%

To solve this e9pression, all we need is to fnd the value of 4
%x

Aow letJs look back at our original equation. If 4
4x
' 1+**, we can fnd the value of
4
%x
b! taking the square root of both sides of the equation. Taking the square root
of an e9ponential e9pression is tanta"ount to halving its e9ponent.
.ince the question asks for 4
%x
E4
%
, the answer is 4*E4
%
, which si"plifes to 4*E1+ or
5E%.
The correct answer is 0
1/.
$15
x
) 15
x)1
&

' 15
y
4
y
Q15
9
) 15
x
$15
1
&R

' 15
y
4
y
$15
x
&$1 ) 15& ' 15
y
4
y
$15
x
&$1+&

' 15
y
4
y
$,
x
&$5
x
&$%
4
& ' $,
y
&$5
y
&$%
%y
&
.ince both sides of the equation are broken down to the product of pri"e bases, the
respective e9ponents of like bases "ust be equal.
%y ' 4 so y ' %.
x ' y so x ' %.
The correct answer is 7
1-.
In proble"s that involve e9ponential e9pressions on both sides of the equation, it is
i"perative to rewrite the bases so that either the sa"e base or the sa"e e9ponent
appears on both sides of the equation. 3ere, we can get a co""on base b! replacing the
- with ,
%
.
,
m
,
m
,
m
' -
n
,
m
,
m
,
m
' $,
%
&
n
,
,m
= ,
%n
.ince the bases are the sa"e, the e9ponents "ust be equal.
,m ' %n
m/n ' %E,
The correct answer is =
5*.
$1& .B88I4I<AT# e can rewrite this equation in a base of ,# ,
b ) %
' ,
5
, which "eans
that b ) % ' 5 and therefore b ' ,.
e can plug this value into the equation a ' ,
b ( 1
to solve for a.
$%& .B88I4I<AT# e can set the right side of this equation equal to the right side of the
equation in the quesiton $both sides equal a&.
,
b ( 1
' ,
%b ( 4
, which "eans that b ( 1 ' %b ( 4 and therefore b ' ,. e can plug this
value into the equation a ' ,
b ( 1
to solve for a.
The correct answer is 0
%1.
@ecogniFe here the basic for" $x (y&
%
, which equals x
%
6 %xy ) y
%
.
corresponds here to x, and corresponds to y.
.o the e9pression can be si"plifed to#
Bnder the radical, recogniFe the basic for" $a ) b&$a ( b&, which equals a
%
( b
%
.
The e9pression can be further si"plifed to#
The correct answer is 4
%%.
The ke! to this question is to recogniFe the co""on algebraic identit!#
a
%
( b
%
' $a ) b&$a ( b&
In this question, the a ter" is xE, and the b ter" is %Ey, which "akes the identit! fro"
the question equal to#
x
%
E- ( 4Ey
%
' $xE, ( %Ey&$xE, ) %Ey& ' 1%
$1& .B88I4I<AT#
.ubstituting the infor"ation fro" this state"ent into the equation fro" the question#
$xE, ( %Ey&$xE, ) %Ey& ' 1%
$xE, ( %Ey&$+& ' 1%
xE, ( %Ey ' %
e now have two equations with x and y#
xE, ) %Ey ' + $fro" this state"ent&
xE, ( %Ey ' % $fro" the substitution above&
4o"bine the two equations $b! adding&, then si"plif!#
$%&$xE,& ' / $the y ter"s cancelled&
xE, ' 4
x ' 1%
$%& .B88I4I<AT#
.ubstituting the infor"ation fro" this state"ent into the equation fro" the question#
$xE, ( %Ey&$xE, ) %Ey& ' 1%
$%&$xE, ) %Ey& ' 1%
xE, ) %Ey ' +
e now have two equations with x and y#
xE, ( %Ey ' % $fro" this state"ent&
xE, ) %Ey ' + $fro" the substitution above&
4o"bine the two equations $b! adding&, then si"plif!#
$%&$xE,& ' / $the y ter"s cancelled&
xE, ' 4
x ' 1%
The correct answer is 0
%,.
e can deter"ine the value of $a ) b&
%
in one of three wa!s# b! fguring out the su" of a
and b, b! deter"ining what a and b are separatel!, or b! deter"ining the value of a% )
%ab ) b% $which is the quadratic for" of our product of factors&.
$1& IA.B88I4I<AT# 7fter "ultipl!ing both sides b! b we can deter"ine that ab ' 15, but
we know nothing else.
$%& IA.B88I4I<AT# If we 8>I1 $a ( b&% we can learn that a% ( %ab ) b% ' 4. 7lternativel!,
this state"ent also indicates that $a ( b& ' % or 6%. 3owever, neither of these
"anipulations allow us to deter"ine the su" of a and b, the respective values of a and b
individuall!, or the value of a% ) %ab ) b%.
$1& 7A0 $%& .B88I4I<AT# .tate"ent 1 tells us ab ' 15. If we substitute this into the
quadratic equation fro" the second state"ent, we can deter"ine the value of a% ) b% in
the following "anner#
a% ( %$15& ) b% ' 4
a% ( ,* ) b% ' 4
a% ) b% ' ,4
If we know the value of a% ) b%, and the value of ab we can deter"ine the value of a%
) %ab ) b%.
a% ) %ab ) b% '
a% ) b% ) %ab =
,4 ) %$15& ' +4
The correct answer is 4
/".
The equation x
%
y
%
' 1/ ( ,xy is reall! a quadratic, with the xy as the variable.
x
%
y
%
) ,xy ( 1/ ' *
$xy ) +&$xy ( ,& ' *
xy ' , or 6+
3owever, we are told that x and y are positive so xy "ust equal ,.
Therefore, x ' ,Ey and x
%
' -Ey
%
.
7lternativel!, this is a WI4 $variable in choice& and can be solved b! plugging nu"bers. If
we plug a value for y and fnd the corresponding value of x, we can check the answers to
see which one "atches the value of x.
1ooking at the values , and 1/ in the equation, a y value of , "akes sense.
x
%
$,&
%
' 1/ ( ,$x&$,&
-x
%
' 1/ ( -x
-x
%
( -x ) 1/ ' *
x
%
( x ) % ' *
$x ) %&$x ( 1& ' *
x ' 1, 6%
=ut since x cannot be negative, x ' 1
If we plug y ' , into each of the answer choices, $4& and $0& both give an x value of 1.
$7& 1E,
$=& %
$4& 1
$0& 1
$<& 4
e "ust now plug another value of y to decide between $4& and $0&. Blti"atel!, onl! $0&
represents the correct value each ti"e.
The correct answer is 0
%5.
This equation can be "anipulated into a quadratic equation b! squaring both sides#
y
%
' ,y ) 4
y
%
( ,y ( 4 ' *
This quadratic equation can be factored to
$y ) 1&$y ( 4& ' *
There are two possible solutions for y# 61 and 4. The product of the two possible solutions
for y is 64.
The correct answer is 7
%+.
e know that the su" of the cubes of a and b is /# a
,
) b
,
' /. e also know that a
+
(
b
+
' 14. Bsing our knowledge of the quadratic te"plate for the di;erence of two
squares,
x
%
( y
%
' $x ) y&$x ( y&, we can rewrite a
+
( b
+
' 14 as follows#
$a
,
&
%
( $b
,
&
%
' 14
$a
,
( b
,
&$a
,
) b
,
& ' 14
.ubstituting for a
,
) b
,
gives#
$a
,
( b
,
&$/&

' 14
$a
,
( b
,
& ' 14E/ ' :E4
The correct answer is 0
%:.
>ne wa! to answer this question is to substitute 1Ex for x and 1Ey for y in the e9pression,
then si"plif! the resulting e9pression.
Pultipl! the nu"erator and the deno"inator b! xy to eli"inate the fractions.
.ince this is not one of the answer choices, it is necessar! to
si"plif! further. ith the knowledge that y ) x ' x ) y and y ( x
' 6$x ( y&, it can be stated that
The correct answer is 7
%/.
hen a proble" involves variables raised to the fourth power, it is often useful to
represent the" as a square of another square, since this approach will allow us to
appl! "anipulations of squares. 7lso note that since we are dealing with high
e9ponents, the approach of plugging nu"bers would prove ti"e6consu"ing and
prone to error in this case.
Therefore, let2s use algebra to solve this proble". Aote that -x
4
( 4y
4
' $,x
%
&
%
( $%y
%
&
%
. e
can represent this e9pression using the for"ula for the di;erence of two squares# -x
4
(
4y
4
' $,x
%
&
%
( $%y
%
&
%
' $,x
%
) %y
%
&$,x
%
( %y
%
&.
1etJs use this shortcut to si"plif! the equation#
-x
4
( 4y
4
' ,x
%
) %y
%

$,x
%
) %y
%
&$,x
%
( %y
%
& ' $,x
%
) %y
%
&
7t this step, our frst instinct "a! be to divide both sides of the equation b! $,x
%
) %y
%
&.
@e"e"ber that in order to divide both siFes of the equation b! an algebraic e9pression,
we need to know that the value of this e9pression is not equal to Fero, since dividing b!
Fero results in an undefned outco"e. =! looking again at the question, we see that 9 and
! are both non6Fero integers, so $,x
%
) %y
%
& cannot be equal to Fero. Therefore, we can
indeed si"plif! the equation further b! dividing out the $,x
%
) %y
%
& fro" each side#
$,x
%
( %y
%
& ' 1
,x
%
' %y
%
) 1
x
%
' $%y
%
)1&E,
Aote that we could also have found the correct answer b! plugging in nu"bers at this
stage $since we have eli"inated the high e9ponents in the equation&. 8or e9a"ple, if we
plug in ! ' % to the equation $,x
%
( %y
%
& ' 1, we see that x
%
' ,. Aow we can plug ! ' %
into each of the answer choices to fnd that onl! 4 also gives us x
%
' ,.
1Ex )
1Ey
1Ex (
1Ey xy$1Ex )
1Ey!
xy$1Ex (
1Ey!
'

$y ) x&
$y ( x&
$y + x&
$y x&
' 6
$x ) y&
$x ( y&
8inall!, if we are careful, we "ight also see that answer choices 7 and = cannot be
correct because the! are negative, and no non6Fero integer squared can equal a negative
nu"ber.
The correct answer is 4
%-. To fnd the ratio of r to s, we need to be able to solve <IT3<@ for rEs >@ for r and s
independentl!.
$1& IA.B88I4I<AT# The equation provided in state"ent 1 cannot be rewritten in the for"
rEs ' so"e value.
$%& IA.B88I4I<AT# The equation provided in state"ent % can be si"plifed as follows#
r% ( s% ' :
$r ) s&$r ( s& ' :.
3owever, this cannot be rewritten in the for" rEs ' so"e value.
$1& 7A0 $%& .B88I4I<AT# e can substitute the infor"ation fro" state"ent $1& in the
equation fro" state"ent % as follows#
$r ) s&$r ( s& ' :.
$:&$r ( s& ' :.
r ( s ' 1.
7dding this equation to the equation fro" the frst state"ent allows us to solve for r.
$r ( s ' 1&
)
$r ) s ' :&
%r ' /
Thus, r = 4. If r is 4, then s "ust be ,. The ratio of r to s is 4 to ,.
The correct answer is 4
,*.
z is the di;erence between the nu"ber of "en and the nu"ber of wo"en in the choirK
hence z = x y. In order to answer the question Gwhat is zMH, we need to be able to
deter"ine either the value of the quantit! x y, or the values of both x and y fro" which
quantit! x y can be co"puted.
$1& .B88I4I<AT# 7fter adding - to both sides of the equation, we get xX ( %xy ) yX ' -.
.ince we are interested in the variables x and y, it would be helpful to rearrange the
e9pression x" ( %xy ) yX into an e9pression that contains ter"s for x and y individuall!.
This suggests that factoring the e9pression into a product of two su"s is in order here.
.ince the coe5cients of both the xX and the y" ter"s are 1 and the coe5cient of the xy
ter" is negative, the "ost logical frst guess for factors is $x ( y&$x ( y& or $x ( y&X. $e
can quickl! confr" that these are the correct factors b! "ultipl!ing out $x ( y&$x ( y& and
verif!ing that this is equal to xX ( %xy ) yX.& 3ence, we now have $x y&" ' - or x y ' ,
or 6,. .ince the sti"ulus states that z or x y is a ph!sical quantit! $Gthere are z "ore
"en than wo"enYH&, the onl! answer that "akes logical sense is x y ' ,.
$%& IA.B88I4I<AT# 7fter adding %%5 to both sides of the equation, we get xX ) %xy ) yX '
%%5. .ince we are interested in the variables x and y, it would be helpful to rearrange the
e9pression x" ) %xy ) yX into an e9pression that contains ter"s for x and y individuall!.
This suggests that factoring the e9pression into a product of two su"s is in order here.
.ince the coe5cients of both the xX and the y" ter"s are 1 and the coe5cient of the xy
ter" is positive, the "ost logical frst guess for factors is $x ( y&$x ( y& or $x ( y&X. $e can
quickl! confr" that these are the correct factors b! "ultipl!ing out $x ( y&$x ( y& and
verif!ing that this is equal to xX 6 %xy ) yX.& 3ence, we now have $x + y&" ' %%5 or x + y
' 15 or 615. <ven if we could pinpoint the value of x + y to one of those two values, this
knowledge would not give us an! insight as to the value of the quantit! x 6 y, or the
values of x and y individuall!.
The correct answer is 7
,1.
In this proble", we are given the infor"ation to set up the following three equations with
three unknowns#
x ' zE4
x + y + z ' %+
y ' %z
Bsing the "ethod of substitution, we can now solve for each of the three unknowns.
.ince the frst equation, x ' zE4, is an equation for x in ter"s of z, and the third equation,
y ' %z, is an equation for y in ter"s of z, we can replace x and y in the second equation
b! their equivalent e9pressions as follows#
x + y + z ' $z/4& ) $%z& ) z ' %+
e are left with an equation with just one unknown z. e can now solve for F#
$z/4& ) $%z& ) z ' %+
zE4 ) /zE4 ) 4zE4 ' 1,zE4 ' %+
zE4 ' %
z ' /
Aow that we know z, we can easil! solve for y, then co"pute y + z $note# we can also
solve for x, but since we are not interested in the value of x there is no need to do so&#
y ' %z ' %$/& ' 1+
y + z ' 1+ ) / ' %4
.ince the largest factor of an! nu"ber is the nu"ber itself, the largest factor of the su"
of y and z is %4.
The correct answer is <
,%.
.olve the original equation for b#
% ) 5a ( bE% ' ,c
% ) 5a ( ,c ' bE%
4 ) 1*a ( +c ' b
Znowing the value of 1*a ( +c will allow us to calculate the value of b. .o, the rephrased
question beco"es# Ohat is 1*a ( +cMO
$1& IA.B88I4I<AT# Znowing the su" of a and c is not enough to deter"ine the value of
1*a ( +c. 8or e9a"ple, if a ' 1* and b ' ,, then 1*a ( +c ' 1*$1*& ( +$,& ' /%.
3owever, if a ' + and b ' :, then 1*a ( +c ' 1*$+& ( +$:& ' 1/.
$%& .B88I4I<AT# Panipulating the equation gives us the following#
61%c ' 6%*a ) 4
%*a ( 1%c ' 4
1*a ( +c ' %
The correct answer is =
,,.
$1& IA.B88I4I<AT# e can start "anipulating this equation b! "ultipl!ing both sides b!
xy. 3owever, we see that the value of xy depends on the values of x and y.
The value of xy changes according to the values of x and y.
$%& .B88I4I<AT# If we "anipulate this equation and solve for xy, we co"e up with a
distinct value for xy.
The correct answer is =
,4.
To si"plif! a radical in the deno"inator of a fraction, !ou "ust "ultipl! the deno"inator
b! so"ething that will cause the radical to disappear. Uou "ust also "ultipl! the
nu"erator b! this sa"e value so as not to change the value of the fraction. $In e;ect, b!
"ultipl!ing the nu"erator and the deno"inator b! the sa"e value, !ou are "ultipl!ing
the entire fraction b! 1.&

hat will cause the radical in deno"inator to disappearM Pultipl! the
deno"inator b! its co"ple"ent, , as follows#



The correct answer is =
35.
$1E5&["\$1E4&[1/'$1E5["&\$1E%[,+&'1E$5["\%\%[,5&'1E%$5["\%[, 5&
1E%$5["\%[,5&' 1E%$1*&[,5, "eans that 5["\%[,5'1*[,5.
>bviousl!, " is ,5
,+.
1 \ 4[11 can be e9pressed at the for"at %\1*[n,5[%1 \ 4[115[%1 \ %[%%'%\1*[%1.
.o, n'%1
,:.
e can rewrite ,
11
as ,
,
S ,
,
S ,
,
S ,
%
. .ince 5
%
$or %5& is quite close to ,
,
$or %:&, we
can replace each ,
,
with 5
%
since the question asks us to appro9i"ate.

The e9pression beco"es 5
%/
) $ 5
%
S 5
%
S 5
%
S ,
%
& ' 5
#
or 5
%/
) 5
+
$,
%
& ' 5
#
e can factor out a 5
+
as follows# 5
+
$5
%%
) ,
%
& ' 5
#
.ince ,
%
$or -& is insignifcant co"pared to 5
%%
$a huge nu"ber&, we can appro9i"ate the
e9pression as# 5
+
$5
%%
&. # is appro9i"atel! equal to %/.
7nother wa! to look at this proble" is to realiFe that while ,
11
is a big nu"ber, it pales in
co"parison to 5
%/
. The e;ect of adding ,
11
to 5
%/
will be "uch less than "ultipl!ing 5
%/
b!
another 5 $i.e. 5
%-
&. To prove this, letJs look at two s"aller nu"bers, such as 5
4
$+%5& and
,
,
$%:&. hen !ou add %: to +%5, the su" is "uch closer to 5
4
$+%5& then to 5
5
$,1%5&.
The correct answer is 4.
,/.
The e9pression in the question can be si"plifed#
%
x
) %
x
%
y
'
%$ %
x &
%
y
'
%
x)1

%
!
'%
x6y)1
The question can be rephrased as Gwhat is x ( yMH since that would su5ce to help us
solve the e9pression.
$1& .B88I4I<AT# The state"ent provides us with a value for x ( y.

$%& IA.B88I4I<AT# The state"ent cannot be "anipulated to co"e up with a value for x (
y, nor can it alone provide a value for x and y.
The correct answer is 7.
%#.
e can rephrase the question as Ohat is yMO or since we know that , Ohat is xMO
$1& .B88I4I<AT# 8ro" the question we know that , so x ' y%. 7ccording to the
state"ent, we also know that x ' yx. If we set the two equations equal to one another,
y% ' yx. x then "ust be equal to %. $There is another solution to this equation# =oth x
and y are equal to 1. 3owever, the question ste" states that x is not equal to y so we
can eli"inate this possibilit!.&
$%& .B88I4I<AT# If we take the cube root of both sides of the equation x, ' /, we fnd
that x ' %.
The correct answer is 0.
4*.
$1& IA.B88I4I<AT# This gives us a range of possible values for y. The low end of the range
$1E5& is s"aller than :E11, while the high end of the range $11E1%& is greater than :E11.
Thus, we cannot deter"ine whether y is greater than :E11.
$%& .B88I4I<AT# This gives us a range of possible values for y. The low end of the range
$%E-& is s"aller than :E11, and the high end of the range $/E1,& is als$ s"aller than :E11.
Thus, y cannot be greater than :E11.
The correct answer is =.
"1.
The ke! to this question is to recogniFe the two co""on algebraic identities#

$x ) y&
%
' x
%
) %xy ) y
%
$x ) y&$x ( y& ' x
%
( y
%

In this question the x ter" is and the y ter" is , which "akes the two identities
equal to#
If we si"plif! the equation using these identities, we get#
The correct answer is <
4%.
$1& IA.B88I4I<AT# Uou cannot si"pl! divide both sides of the equation b! p to obtain p# '
1. The reason is that !ou don2t know whether or not p is Fero 66 and re"e"ber, !ou are
not allowed to divide b! FeroL Instead, !ou "ust factor this equation.
8irst, subtract p fro" both sides to get# p#p ( p ' *.
Then, factor out a co""on p to get# p$p# ( 1& ' *. This "eans that either p# ' 1 or p '
*.
$%& IA.B88I4I<AT# The sa"e process applies here as with state"ent $1&. @e"e"ber, !ou
should never divide both sides of an equation b! a variable that could be Fero.
8irst, subtract # fro" both sides to get# #p# ( # ' *.
Then, factor out a co""on # to get# #$p# ( 1& ' *. This "eans that either p# ' 1 or # '
*.

$1& 7A0 $%& IA.B88I4I<AT# Together we still donJt have enough infor"ation to
solve. <ither p# ' 1 or both p and # are *.
The correct answer is <
4,.
Aotice that the identit! in the nu"erator of the original fraction is written in the for"
x
%
y
%
, with x = 4x
%
, and y = -y
%
K to factor, rewrite it as $x + y&$x y&, or $4x
%
+ -y
%
&$4x
%
%
-y
%
&. e can also factor the right side of the frst equation#

e can co"bine this equation with the other equation in the question#

%x ,y ' ,
4 x ) , y ' -
+x ' 1%
x ' %
The correct answer is 4.
44.
.ince f$x& ' ax
4
( 4x
%
) ax ( ,,
f$b& ' ax
4
( 4x
%
) ax ( ,
' ab
4
( 4b
%
) ab ( ,
7A0
f$6b& ' ax
4
( 4x
%
) ax ( ,
' a$6b&
4
( 4$6b&
%
) a$6b& ( ,
' ab
4
( 4b
%
( ab ( ,
Therefore#
f$b& ( f$6b& ' ab
4
( 4b
%
) ab ( , ( $ab
4
( 4b
%
( ab ( ,& ' %ab
7lternativel!, we could have recogniFed that the onl! ter" of the function that will be
di;erent for f$b& than for f$6b& is the Oax.O The other three ter"s are all una;ected b! the
sign of the variable. Pore succinctl!, f$b& ( f$6b& "ust equal ab ( $6ab& ' %ab.
The correct answer is =
45.
e are told that p]# ' p
%
) #
%
( %p#. In order for p]# ' p
%
, the value of #
%
( %p#
"ust equal *. e can solve this as follows#
#
%
( %p# ' *
#$# ( %p& ' *
The solution that would work for all value of p is if # ' *.
In plugging # ' * back into the original function, we get#
p&# ' p
%
) *
%
6 %p$*& ' p
%
.
The correct answer is 4
"5.
The e9pression in the question can
be rewritten as
1
t
%
'
,
.
$1& .B88I4I<AT# This state"ent can be rewritten as follows#
Therefore, t,'% ' ,+. The onl! positive integers that satisf! this
e9pression are t ' 1 and ' = +. .ince we know the values of t and ',
we can solve the e9pression in the question.
$%& IA.B88I4I<AT# This state"ent can be rewritten as follows#
There are "an! possible values for t and '. 8or e9a"ple, t
could be % and ' could be 1%. 7lternativel!, t could be 1 and ' could be +. .ince, there
are "an! possibilities for t and ', we are not able to solve the e9pression in the question.
The correct asnwer is 7.
4:.
The ke! to this proble" is recogniFing that the e9pression a ( b can be factored as the
di;erence of two squares $x
%
( y
%
&, where .
The left side of the equation can be factored as follows#
The correct answer is 4
"&.
$here is no useful rephrase of the question, so the best approach here is to analyze the statement to see hat
they tell us about the values of a, b, c, and d.
&tatement - tells us that bLdL ) 4'aLdL(. If e divide both sides by dL, e are left ith bL ) 4aL. Aemember
that to find the factorial value of an integer, you multiply that integer by every positive integer smaller than it.
&ince bL is 4 times greater than aL, it must be true that bL ) 4 x a x 'a , -( x 'a , +(... &ince bL is a factorial
product and cannot have more than one 4 as a factor, it must be true that bL ) 4 x % x + x -. $herefore, a ) %
and b ) 4. Cut this tells us nothing about c or d. Insufficient.
&tatement + tells us that ?.'bLcL( ) 'bLdL(. If e divide both sides by bL, e are left ith ?.cL ) dL. &ince dL is
?. times greater than cL, dL could equal ?.L 'i.e., ?. x *2 x *>...(, and therefore d ) ?. and c = *2. #r dL
could equal 'cL('%('4('*(, in hich case cL must be +L and c ) + and d ) *. Insufficient.
If e pool the information from both statements, hoever, e see that ?.'bLcL( ) 4'aLdL(, hich yields -*'bL
cL( ) 'aLdL(. If e try this equation ith a ) %, b ) 4, c ) *2, and d ) ?., e get -*'4L*2L( ) '%L?.L( or ?.'%L
*2L( ) '%L?.L(, hich is the same as %L?.L ) %L?.L. &o these four values are possible.
If e try the equation ith a ) %, b ) 4, c ) +, and d ) *, e get -*'4L+L( ) '%L*L( or '%('*('4L+L( ) '%L*L(,
hich is the same as *L%L ) %L*L. &o these four values are possible as ell.
&ince the value of c can be either + or *2 and the value of d can be either * or ?., e cannot anser the
1
t
,
'
%
'
1
,+
t$'
61
&
'
t
'
'
1
+
question definitively.
$he correct anser is H.
42.
$he simplest approach to this problem is to pic! numbers. Bet's say that x ) -. We can plug in - for x in :
:nd e can plug in - for x in :
$herefore, ill be equal to . If e evaluate each choice by plugging in -
for x, the only one to give as an anser is ::
:lternatively, e can solve algebraically.
4irst, let's calculate the value of : . We can simplify this in terms of :
"o let's calculate the value of : . We can simplify this in terms of :
We can no see that is equal to the folloing:
$he correct anser is :.
50.
4irst, simplify the numerator by letting x = ab. $hen the numerator can be simplified as follos:
&ubstituting ab bac! in for x, the original equation no loo!s li!e this:
In order for the fraction to have a value of ., the numerator must have a value of ..
$hus, ab can be equal to ., ,?, or %. Doever, since e are told that a and b are both nonzero integers, ab
cannot be . and it must be equal to ,? or %. $herefore, a and b must be integer factors of ,? or %. $hus it
ould appear that:
b can be equal to +, if a ) ,%
b can be equal to %, if a ) ,+
b can be equal to %, if a ) -
Doever, a cannot be equal to ,+ or -, since this ould ma!e the denominator equal to . and leave the
fraction undefined. $his leaves one option: a ) ,% and b ) +. $he correct anser is : 'I only(: the variable b
can be equal to +, not % or 4.
51.
This proble" can be solved either algebraicall! or b! picking nu"bers.
If the greater of the two integers is x, then the two integers can be e9pressed as x ( 1
and x. The su" of the reciprocals would therefore be
1
x ( 1
)
1
x
5%.
8irst, look at state"ent $1& b! itself.
y ' x $x 6 ,& $x ) ,&
0istributing the right side of the equation#
y ' x $x
%
( -&
y ' x
,
( -x
.ubtract ever!thing on the right fro" both sides to get#
y ( x
,
) -x ' *, which al"ost looks like the e9pression in the question.
To "ake the left side of the equation "atch the question, subtract /x fro" both sides#
! ( x
,
) x ' 6/x
e would be able to answer the question if onl! we knew the value of x, but that
infor"ation is not given. .tate"ent 1 is not su5cient.
.econd, look at state"ent $%& b! itself.
y ' 65x
.ince the question asks about a co"plicated e9pression of 9Js and !Js, the si"plest wa!
to see if state"ent $%& is su5cient is to tr! to "ake one side of the equation in state"ent
$%& "atch the question, then tr! to si"plif! the other side of the equation to a single
value. .ince the question asks about the value of y ) x
,
) x, and state"ent $%& has ! on
the left side of the equation, add the G"issingH x
,
) x to both sides of the equation in
state"ent $%&.
y ) $x
,
) x& ' 65x ) $x
,
) x&
y ) x
,
) x ' x
,
6 4x
y ) x
,
) x ' x $x
%
( 4&
e would be able to answer the question if onl! we knew the value of x, but that
infor"ation is not given. .tate"ent % is not su5cient.
8inall!, look at both state"ents together.
.ince both give e9pressions for y, set the right sides of each state"ent equal to each
other#
65x ' x $x ( ,& $x ) ,&
65x ' x $x
%
( -&
659 ' x
,
( -x
* ' x
,
( 4x
* ' x $x
%
( 4&
* ' x $x ( %&$x ) %&
.o, there are three solutions for x# ^*, %, or 6%_. 7t frst, the state"ents together "ight
x
x$x ( 1&
)
x ( 1
x$x ( 1&
%x ( 1
x
%
( x
see" insu5cient, since this !ields three values. 3owever, the question is not asking the
value of x, rather the value of y ) x
,
) x. It is a good idea to fnd the value of y for each x
value, then solve for the e9pression in the question.
hen x ' *, y ' * and y ) x
,
) 9 ' * ) * ) * ' *
hen x ' %, y ' 61* and y ) x
,
) x ' 61* ) / ) % ' *
hen x ' 6%, y ' 1* and y ) x
,
) x ' 1* 6 / 6 % ' *
The answer "ust be Fero, so the two state"ents together are su5cient. The correct
answer is 4.
5,.
The ke! to solving this proble" is to recogniFe that the two given equations are related
to each other. <ach represents one of the ele"ents in the co""on quadratic for"#
@ewrite the given equation as follows#
Then, notice it2s relationship to the second given equation#
The second equation is in the for" , while the frst equation is in the for"

.ince we know that and that and that we can solve for , which
"ust equal /.
This gives us a third equation# .
7dding the second and third equations allows us to solve for x as follows#
Nlugging this value for x into the frst equation allows us to solve for y + z as follows#
The question asks for the value of x + y + z.
If x = 1% and y + z = %, then x + y + z = 1% ) % ' 14.
The correct answer is <.
54.
If we square both sides of the equation, we get z
%
' +zs ( -s
%
. e can now put the
quadratic in standard for" z
%
( +Fs ) -s
%
' * and factor $z ( ,s&
%
' *. .ince z ( ,s ' *, z
' ,s. This question can also be solved as a WI4 b! plugging a value for the variable s. If
we sa! s ' %, and plug this value into the equation after it was squared, we get#
z
%
' 1%z ( ,+
This can be written in standard for" and factored# $z ( +&
%
' *, which "eans that z ' +.
Aow we can see which answer choice$s& !ield + as the value for z when we plug in s ' %.
>nl! answer choice = works, ,$%& ' +.
The correct answer is =.
55.
5+.
The left side of the equation fts the for" a
%x
( b
%x
, which can be factored as
$a
x
) b
x
&$a
x
( b
x
&.
Thus, we can rewrite the equation as >@

.ince , one wa! to solve for is to fnd the value of
.
7nother wa! to solve for is to fnd the value of a and b.
$1& .B88I4I<AT# e can subtract root , fro" both sides of the equation to obtain
' %.
$%& IA.B88I4I<AT# e can plug a ' +4 into the original equation to solve for b.
$+4&
%E,
( b
%E,
' 1%
1+ ( b
%E,
' 1%
b
%E,
' 4
b ' 4
,E%
' )E6 /
ith a single value for a but two values for b, there are two solutions to the question.
The correct answer is 7, .tate"ent $1& 71>A< is su5cient to answer the question, but
state"ent $%& alone is not.
57.
To fnd the value of a6b we "ust either fnd the values of a and b and subtract the" or
so"ehow "anipulate an equation so that we can solve directl! for the co"bined
e9pression a6b.
.tate"ent $1& can be rewritten as#

.ince a, b, x and y are all positive integers, a6b "ust be an integer $although not
necessaril! a positive one&. It "a! be easier to write this state"ent out in words# an
integer $x! raised to so"e integer power $a6b& "ust equal three.
.ince , is a pri"e nu"ber, it "ust follow that the base x is also ,. If x were so"ething
other than ,, there would be no wa! of raising it to an integer power and co"ing up
with ,. <ven the base -, which is co"posed of onl! ,2s, would need to be raised to a
fractional e9ponent $i.e. square root ' power of 1E%& to co"e up with ,.
If x "ust be ,, we can set up the following equation# ,
a6b
' ,
1
. It follows that a6b ' 1 and
state"ent $1& is .B88I4I<AT. The answer "ust be 7 or 0.
.tate"ent $%& can be dealt with in a si"ilar "anner#
Aotice, however, that the e9pression !
a6b
now equals 4, which is not a pri"e nu"ber.
=ecause 4 can be e9pressed as 4
1
or %
%
, the base y and the e9ponent a6b do not have
f9ed values. .tate"ent $%& is IA.B88I4I<AT and the correct answer is 7.
58.
hen a bino"ial is e9panded, the nu"ber of ter"s is alwa!s one "ore than the
e9ponent of the bino"ial. 8or e9a"ple,
. There are three ter"s and three is one "ore than two, the
e9ponent.
In this case, we have si9 ter"s, which "eans the value of 9 "ust be 5. e can now
rewrite the e9pression in the question b! substituting 5 for 9 as follows#
The question asks for the value of yz. To answer this we need to fgure out the values of
the coe5cients y and z.
1etJs consider the coe5cient in an easier e9pression such as

Aotice that the coe5cient % represents the nu"ber of wa!s that one a and one b can be
"ultiplied together# either ab or ba.
This is akin to counting the nu"ber of per"utations of % unique ele"ents# %L ' %.
.i"ilarl!, in , the coe5cient , represents the nu"ber of
wa!s !ou can "ultipl! together two of the sa"e ter" and one of the other# aab, aba, baa
or bba, bab, abb.
This is akin to counting the nu"ber of per"utations of % identical ele"ents and 1 unique
one#
In the case at hand, , we can fgure out the
value of the coe5cient y, b! counting the nu"ber of per"utations of 4 as and 1 b $or 4
bJs and 1 a&#
, so ! ' 5.
To fgure out the value of the coe5cient of z, we need to count the nu"ber of
per"utations of , a2s and % b2s $or % bJs and , as&#
, so F ' 1*.
Therefore, the value of yz is $5&$1*& ' 5*. The correct answer is <.
5.
e are given an equation with two variables and asked to fnd the product of the
variables. 7t frst glance, it "a! look i"possible to solve this equation for 9 and !, since
we have two variables and onl! one equation. >n top of that, the variables appear onl!
as e9ponents. 7nd, to pile it on, each answer choice has so "an! factors that it would
be totall! i"practical to start b! plugging in nu"bers. 3owever, using a co"bination of
algebra and logic, we can fgure out the values of 9 and ! and then fnd their product.
8irst, let2s rewrite the given equation so that all the variables are "oved to one side#
Aow, so"e logic is necessar! to fnish up.
e know that that both ter"s, and , are positive.
.ince "ust equal 5, we also know that and "ust each be less than
5.
e can now list all the possibilities, b! testing s"all integer values for !. e onl! have
to test a few values because we know that "ust be less than 5, which "eans that
! can onl! be 1, % or ,. $If ! is 4, then would be /, which is greater than 5.&
If ! ' 1, then , which "eans that "ust equal 4 $re"e"ber, the
su" of the two ter"s "ust be equal to 5&. 3owever, since 9 and ! are positive integers,
there is no wa! to "ake equal to 4.
If ! ' %, then , which "eans that "ust equal ,. 3owever, since 9 and
! are positive integers, there is no wa! to "ake equal to ,.
If ! ' ,, then , which "eans that "ust equal 1. This is possible.
Bsing ! ' ,, we can solve for 9 as follows#
The onl! possible solution is ! ' , and 9 ' 4. Therefore, 9! ' $4&$,& ' 1%. The correct
answer is <.
6!.
8ro" the proble" state"ent, we know that .
e also know that the digits b c e and f are integers fro" * to - and that the digits a
and ( are integers fro" 1 to - $the! cannot be * since the! are in the hundreds place&.
8or the state"ent above to be true, "ust equal , and .
Therefore, .
.ince the onl! di;erence between abc and (ef is in the units digits, the di;erence
between these three6digit nu"bers is equal to , or 1.
The correct answer is 7.
61.
In this t!pe of proble", the easiest thing to do is to e9press both sides of the equation
in ter"s of pri"e nu"bers. The left side of the equation is alread! e9pressed in ter"s
of pri"e nu"bers, so we need to start b! rewriting the right side of the equation in
ter"s of pri"e nu"bers#
Thus, the given equation can be rewritten as follows#
`uite a bit is revealed b! putting the equation in this for". The right side of the
equation, which we will call the GtargetH, is co"prised onl! of %Js and 5Js. 1ooking at
the left side of the equation, we see that we have x nu"ber of %Js and y nu"ber of 5Js
along with so"e factor z. This unknown factor z "ust be co"prised of onl! %Js, onl!
5Js, so"e co"bination of %Js and 5Js, or it "ust be 1 $i.e. with no pri"e factors&. This is
because an! other pri"e co"ponents of F would !ield a product that is di;erent fro"
the target.
The question asks us to solve for xy, which we can certainl! do b! deter"ining the
values of both x and y. .ince x and y si"pl! tell us the nu"ber of %Js and 5Js,
respectivel!, that will be contributed toward the product on the left side of the equation
$e.g., if x ' % and y ' ,, there are two %Js and three 5Js toward the product&, we can
look at this question in a slightl! di;erent light. The onl! other contributor to the fnal
product on the left side is z. If we knew how "an! %Js andEor 5Js that z contributed to
the product, this would be enough to tell us what x and y are. 7fter the inclusion of z,
an! surplus or defcit of %Js would have to be covered b! x and an! surplus or defcit of
5Js would have to be covered b! y. In other words, the question what is xy can be
rephrased as what is zM
In state"ent $1& we are told that z ' %*, which is su5cient to answer our rephrased
question. Cust to illustrate, this state"ent "eans that z provides the product on
the left side of our equation with two additional %Js and an additional 5, since
. e can use this infor"ation to solve for x and y as follows#

In state"ent $%& we are given the nu"ber of %Js contributed b! the e9pression %
x
on
the left side of the equation. To hit the target, z "ust contain e9actl! two %Js, since
. =ut what about 5JsM Is the e9pression 5
y
the onl! source of 5Js or is z
co"posed of 5Js as wellM e have no wa! of knowing so we cannot fnd the value of z
$or y&.
The correct answer is $7&# .tate"ent $1& alone is su5cient, but state"ent $%& alone is
not su5cient.
62.
>ne of the "ost e;ective wa!s to begin solving proble"s involving e9ponential
equations is to break down bases of the e9ponents into pri"e factors and co"bine
e9ponents with the sa"e base. 8ollowing this approach, be sure to si"plif! each
state"ent as "uch as possible before arriving at the conclusion, since di5cult proble"s
with e9ponents often result in unobvious outco"es.
$1& IA.B88I4I<AT# hile this state"ent gives us the value of x, we know nothing about y
and cannot deter"ine the value of x
y
.
$%& .B88I4I<AT#
$1%/
x
&$+
x + y
& ' $4/
%x
&$,
%x
&
$%
:
&
x
$%

S ,&
x + y
' $%
4
S ,&
%x
$,
%x
&
$%
:x
&$%
x ) y
&$,
x ) y
& ' $%
/x
&$,&
%x
$,
%x
&
$%
/x + y
&$,
x + y
& ' $%
/x
&$,&
%x % x
$%
/x
&$

%
y
&$,
x
&$,
y
& ' $%
/x
&$,&
x
$

%
y
&$,
y
& ' 1
$%

S ,&
y
' 1
+
y
' 1
y ' *
.ince y ' * and x is not equal to Fero $as stated in the proble" ste"&, this infor"ation is
su5cient to conclude that x
y
' x
*
' 1.
The correct answer is =.
5%.
)n order to evaluate the function f+n-, simply substitute the value of n for every instance of n
on the rightGhandGside of the definition of the function. :or e*ample, for n , ", f+"- ,
f+%- C ". <ote that the value of f+n- is dependent on the value of f+n C 1-. Therefore, in
order to find f+"-, we must ;now f+%-. 'o one way of rephrasing the question is
WWhat is the value of f+%-QX
Mowever, let6s suppose we don6t ;now f+%- but we ;now f+/-. 'ince, f+%- , f+/- C %,
we can calculate the value of f+%- from f+/-, then f+"- from f+%-. Aontinuing this logic,
if we ;now the value of f+1-, we can calculate the value of f+/- from f+1-, then f+%-
from f+/-, and then f+"- from f+%-. )t is apparent that if we ;now the value for f+i-
where i is any integer less than ", we can eventually get to the value of f+"- by
successive calculating f+n- for increasing n6s.
We can also rearrange the equation f+n- , f+n C 1- C n to f+n C 1- , f+n- 4 n. 'o if we
;now f+5-, then f+5 C 1- or f+"- , f+5- 4 5. Mence, given f+5-, we can calculate f+"-.
Bsing similar logic as above, if we ;now f+5-, we can calculate the value of f+5- from
f+5-, then f+"- from f+5-. We can see that we ;now the value of f+i- for any integer i
greater than ", we can eventually get to the value of f+"- by successively calculating
f+n C 1- for decreasing n6s.
Therefore, if we ;now the value of f+i- for any one specific value of i, we can get to
the value of f+"-S hence, the question can be restated as What is the !al"e #$ $%i& $#'
any s(e)i$i) inte*e' i+,
+1- 'B::)A)?<T. 'ince we are given the value of f+i- for the specific integer i , %, it
follows that f+"- can be calculated.
+/- 'B::)A)?<T. 'ince we are given the value of f+i- for the specific integer i , 5, it
follows that f+"- can be calculated.
The correct answer is $.
5".
'ince we ;now the value of YG!Y , %, we can plug p , G! into our formula
+G!-
%
a + +G!-3 C 1 , %
G%"%a C !3 , %
G%"%a C !3 , "
We are as;ed to solve for Y!Y. )f we plug ! into our formula, we get
+!-
%
a 4 +!-3 < 1 , Q
%"%a 4 +!-3 C 1 , Q

To figure this out, we would need to ;now the value of %"%a 4 !b.

:rom the first equation we ;now that G%"%a < !3 , ". .y multiplying both sides by negative
one, we see that %"%a 4 !3 , G".

%"%a + !3 C 1 , Q
G" C 1 , G5

The correct answer is ?.
55.
)t helps to recogniJe that this is a quadratic equation problem presented as a function
problem. What we are essentially being told is that 5 and G% are the two Jeros for the
equation x
/
4 3x 4 (. )n other words, 5 and G% are the two solutions to the equation x
/
4
3x 4 ( , 0. .ecause solutions are always the opposites of the factor numbers, we
;now that our equation in factored form is
+x C 5-+x 4 %- , 0
which, when :=)2ed, becomes
x
/
C %x C 1& , 0
'o 3 , G%, and ( , G1&. Therefore, 3 4 ( , G/1.
(lternatively, we can find the values of 3 and ( by using substitution.
)f f+5- , 0, then
5
/
4 3+5- 4 ( , 0
53 4 ( , G%5
)f f+G%- , 0, then
+G%-
/
4 3+G%- 4 ( , 0
G%3 4 ( , G#
We can combine the two equations and solve. )n this method, we add or subtract the
two equations to eliminate one of the variables. )n this problem, we can use
subtraction to eliminate (.
53 4 ( , G%5
C +G% 3 4 ( , G#-
#3 , G/!
3 , G%
We can substitute G% for 3 into either of the above equations to get ( , G1&. )t follows
that 3 4 ( , G/1.
The correct answer is $.
55.
This problem can be solved algebraically or by plugging in the answer choicesS both methods
are shown below.
%lge3ra
.ecause there are square root signs on both sides of the equation, we can square both sides to
get rid of them, which leaves us with " 4 x
1D/
, x 4 /. x
1D/
is the same thing as , so our ne*t
step is to isolate the radical sign and then square both sides again. =nce we do this, we can
solve for x.
, x C /
x , +x C /-
/

x , x
/
C "x 4 "
0 , x
/
C 5x 4 "
0 , +x C "-+x C 1-
x can equal either " or 1S try each to determine which one is the solution to the original
equation
x , " , , , so x can equal ".
x , 1 , so x cannot equal 1.
=nly x , " wor;s.
Plugging in the ans!ers
'ince the numerical answers represent a possible value for x, we can also plug them into the
equation and see which one wor;s. @emember that we can stop when we find the right
answer and itEs also best to start with answer choice A. )f A does not wor;, we can sometimes
determine whether we want to try a larger or smaller number ne*t, thereby saving some time.
+?- 'ince $ wor;s, the answer can be determined. :(2'?
The correct answer is $.
5!.
The equation in question can be rephrased as follows
x
/
y C 5xy 4 #y , 0
y+x
/
C 5x 4 #- , 0
y+x C %-
/
, 0
Therefore, one or both of the following must be true
y , 0 or
x , %
)t follows that the product xy must equal either 0 or %y. This question can therefore be
rephrased WWhat is yQX
+1- )<'B::)A)?<T This equation cannot be manipulated or combined with the
original equation to solve directly for x or y. )nstead, plug the two possible scenarios
from the original equation into the equation from this statement
)f x , %, then y , % 4 x , % 4 % , 5, so xy , +%-+5- , 1&.
)f y , 0, then x , y C % , 0 C % , G%, so xy , +G%-+0- , 0.
'ince there are two possible answers, this statement is not sufficient.
+/- 'B::)A)?<T )f x
%
O 0, then x O 0. Therefore, x cannot equal %, and it follows
that y , 0. Therefore, xy , 0.
The correct answer is ..

You might also like